You are on page 1of 92

PROMETRIC PRACTICE-LAB TECHNOLOGIST

Microbiology
1. Which of the following bacterial groups is characterized by a thick peptidoglycan layer in their cell walls,
leading to a purple color in the Gram stain?
a) Gram-negative bacteria
b) Gram-positive bacteria
c) Archaea
d) Fungi
Answer: b) Gram-positive bacteria

2. Which of the following is a common sign of a Gram-positive bacterial skin infection?


a) Difficulty breathing
b) Jaundice
c) Localized pain and swelling
d) Frequent urination
Answer: c) Localized pain and swelling

3. Streptococcus pneumoniae is known to cause which of the following conditions?


a) Tuberculosis
b) Pneumonia
c) Malaria
d) Urinary tract infection
Answer: b) Pneumonia

4. What is the primary role of toxins produced by some Gram-positive bacteria during infection?
a) Promote host cell growth
b) Enhance antibiotic effectiveness
c) Damage host cells and tissues
d) Increase oxygen levels in tissues
Answer: c) Damage host cells and tissues

5. Which laboratory technique is used to differentiate Gram-positive from Gram-negative bacteria based on
cell wall structure?
a) DNA sequencing
b) Electron microscopy
c) Gram staining
d) Western blotting
Answer: c) Gram staining

6. In a patient with a suspected Gram-positive bacterial infection, which laboratory finding might indicate an
inflammatory response?
a) Elevated white blood cell count
b) Decreased red blood cell count
c) Low platelet count
d) Reduced serum glucose levels

Page 1 of 92
Answer: a) Elevated white blood cell count

7. What type of laboratory test is performed to determine which antibiotics are effective against a specific
Gram-positive bacterial strain?
a) Blood culture
b) Antibiotic susceptibility testing
c) Serological assay
d) PCR analysis
Answer: b) Antibiotic susceptibility testing

8. Which laboratory test can detect specific toxins produced by Gram-positive bacteria in certain infections?
a) Gram staining
b) Blood culture
c) Toxin detection assay
d) Urinalysis
Answer: c) Toxin detection assay

9. Which of the following is a common symptom of a Staphylococcus aureus infection?


a) High fever
b) Watery diarrhea
c) Skin rash
d) Rapid heartbeat
Answer: c) Skin rash

10. Streptococcus pyogenes, also known as Group A Streptococcus, can cause which of the following
conditions?
a) Influenza
b) Stomach ulcers
c) Streptococcal pharyngitis (strep throat)
d) Meningitis
Answer: c) Streptococcal pharyngitis (strep throat)

11. In Gram staining, what color do Gram-positive bacteria appear after the staining process?
a) Purple or blue
b) Red or pink
c) Green
d) Yellow
Answer: a) Purple or blue

12. Which type of Gram-positive bacteria is known for its rod-shaped appearance?
a) Staphylococci
b) Streptococci
c) Bacilli
d) Cocci
Answer: c) Bacilli

13. What is the primary purpose of antibiotic susceptibility testing in the context of Gram-positive bacterial
infections?

Page 2 of 92
a) To diagnose the infection
b) To determine the bacterial species
c) To assess the patient’s immune response
d) To identify effective antibiotics for treatment
Answer: d) To identify effective antibiotics for treatment

14. Which laboratory test is used to confirm the presence of antibodies produced by the host in response to a
Gram-positive bacterial infection?
a) Blood culture
b) PCR analysis
c) Serological assay
d) Western blotting
Answer: c) Serological assay

15. Which of the following is NOT a common sign of a Clostridium difficile infection?
a) Severe diarrhea
b) Abdominal pain
c) Headache
d) Fever
Answer: c) Headache

16. Which bacterial genus is known for its spherical (round) cell shape and often forms chains or clusters?
a) Staphylococcus
b) Streptococcus
c) Escherichia
d) Salmonella
Answer: b) Streptococcus

17. What type of laboratory test can detect specific bacterial DNA in a clinical sample, aiding in the diagnosis
of a Gram-positive bacterial infection?
a) Blood culture
b) PCR analysis
c) Gram staining
d) ELISA assay
Answer: b) PCR analysis

18. Which Gram-positive bacterial species is commonly associated with food poisoning and can produce toxins
that cause gastrointestinal symptoms?
a) Clostridium difficile
b) Listeria monocytogenes
c) Mycobacterium tuberculosis
d) Escherichia coli
Answer: b) Listeria monocytogenes

19. In a patient with a Streptococcus pneumoniae infection, which respiratory symptom is most commonly
observed?
a) Runny nose
b) Cough and chest pain
c) Nausea and vomiting

Page 3 of 92
d) Joint pain
Answer: b) Cough and chest pain

20. What laboratory finding is typically associated with an elevated immune response to a Gram-positive
bacterial infection?
a) Decreased white blood cell count
b) Low levels of C-reactive protein (CRP)
c) Elevated erythrocyte sedimentation rate (ESR)
d) Decreased antibody production
Answer: c) Elevated erythrocyte sedimentation rate (ESR)

21. Which of the following bacterial genera includes species responsible for skin infections, such as impetigo
and cellulitis?
a) Clostridium
b) Listeria
c) Staphylococcus
d) Mycobacterium
Answer: c) Staphylococcus

22. Which Gram-positive bacterial species is commonly associated with urinary tract infections (UTIs) and is
known for its ability to form biofilms on catheters?
a) Streptococcus pyogenes
b) Clostridium difficile
c) Enterococcus faecalis
d) Mycobacterium tuberculosis
Answer: c) Enterococcus faecalis

23. In Gram staining, what color do Gram-negative bacteria appear after the staining process?
a) Purple or blue
b) Red or pink
c) Green
d) Yellow
Answer: b) Red or pink

24. Which of the following is a potential complication of untreated Streptococcus pyogenes infections, such as
streptococcal pharyngitis (strep throat)?
a) Kidney failure
b) Hearing loss
c) Blindness
d) Osteoporosis
Answer: a) Kidney failure

25. What is the primary mechanism by which Streptococcus pneumoniae causes pneumonia in the lungs?
a) Toxin production
b) Invasion of lung cells
c) Disruption of mucus production
d) Formation of biofilms
Answer: b) Invasion of lung cells

Page 4 of 92
26. Which of the following laboratory tests is used to identify specific antibodies against Gram-positive
bacterial antigens in a patient’s blood sample?
a) Blood culture
b) PCR analysis
c) Serological assay
d) Western blotting
Answer: c) Serological assay

27. Which Gram-positive bacterium is known for its role in causing dental caries (cavities) by forming biofilms
on tooth surfaces?
a) Clostridium difficile
b) Streptococcus mutans
c) Staphylococcus aureus
d) Mycobacterium leprae
Answer: b) Streptococcus mutans

28. Which laboratory technique is used to observe the morphology and arrangement of bacterial cells after
Gram staining?
a) Blood culture
b) Electron microscopy
c) ELISA assay
d) Western blotting
Answer: b) Electron microscopy

29. Which of the following is a common symptom of a Clostridium difficile infection?


a) Skin rash
b) High fever
c) Severe diarrhea
d) Joint pain
Answer: c) Severe diarrhea

30. Which laboratory test is often performed to confirm the presence of Gram-positive cocci in clusters in a
clinical sample?
a) PCR analysis
b) Gram staining
c) Blood culture
d) ELISA assay
Answer: b) Gram staining

31. Which Gram-positive bacterium is responsible for causing tuberculosis, a disease that primarily affects the
lungs?
a) Staphylococcus aureus
b) Streptococcus pneumoniae
c) Mycobacterium tuberculosis
d) Clostridium difficile
Answer: c) Mycobacterium tuberculosis

32. Which of the following signs and symptoms is often associated with a Streptococcus pyogenes (Group A
Streptococcus) skin infection?

Page 5 of 92
a) Frequent urination
b) Nausea and vomiting
c) Fever and sore throat
d) Joint pain and stiffness
Answer: d) Joint pain and stiffness

33. Which laboratory finding may indicate an inflammatory response to a Gram-positive bacterial infection
when measured in a blood test?
a) Elevated white blood cell count
b) Decreased erythrocyte sedimentation rate (ESR)
c) Low levels of C-reactive protein (CRP)
d) Reduced platelet count
Answer: a) Elevated white blood cell count

34. What is the primary function of antibiotic susceptibility testing in the context of Gram-positive bacterial
infections?
a) To diagnose the infection
b) To identify the bacterial species
c) To determine antibiotic resistance
d) To assess patient immunity
Answer: c) To determine antibiotic resistance

35. Which Gram-positive bacterium is commonly associated with foodborne illnesses and can produce heat-
resistant endospores?
a) Listeria monocytogenes
b) Streptococcus pyogenes
c) Clostridium difficile
d) Staphylococcus aureus
Answer: a) Listeria monocytogenes

36. What laboratory technique is used to detect specific bacterial DNA in a clinical sample, allowing for the
diagnosis of a Gram-positive bacterial infection?
a) Blood culture
b) PCR analysis
c) Serological assay
d) Gram staining
Answer: b) PCR analysis

37. Which of the following Gram-positive bacteria is commonly found in the gastrointestinal tract and can
cause urinary tract infections when it enters the urinary system?
a) Clostridium difficile
b) Enterococcus faecalis
c) Staphylococcus aureus
d) Streptococcus pneumoniae
Answer: b) Enterococcus faecalis

38. In Gram staining, what component of the bacterial cell wall retains the crystal violet stain in Gram-positive
bacteria?

Page 6 of 92
a) Lipopolysaccharide
b) Peptidoglycan
c) Outer membrane
d) Capsule
Answer: b) Peptidoglycan

39. What is the primary purpose of serological assays in diagnosing Gram-positive bacterial infections?
a) To identify bacterial species
b) To assess antibiotic susceptibility
c) To detect specific antibodies in patient serum
d) To visualize bacterial morphology
Answer: c) To detect specific antibodies in patient serum

40. Which of the following is NOT a common clinical manifestation of a Staphylococcus aureus infection?
a) Skin abscess
b) Cellulitis
c) Pneumonia
d) Frequent urination
Answer: d) Frequent urination

Major pathogens and An aerobic bacteria


1. What is a major pathogen in microbiology?

a) Virus

b) Bacteria

c) Fungus

d) All of the above

2. Which of the following is considered a major pathogen that causes tuberculosis?

a) Streptococcus pneumoniae

b) Escherichia coli

c) Mycobacterium tuberculosis

d) Salmonella enterica

3. Which virus is responsible for causing the common cold, a major respiratory infection?

a) Influenza virus

b) Rhinovirus

c) Human papillomavirus

Page 7 of 92
d) Hepatitis B virus

4. What is the primary pathogen responsible for causing malaria?

a) Plasmodium falciparum

b) Vibrio cholerae

c) Staphylococcus aureus

d) Candida albicans

5. Which of the following bacteria is a major pathogen responsible for food poisoning?

a) Listeria monocytogenes

b) Clostridium botulinum

c) Campylobacter jejuni

d) Streptococcus pyogenes

6. Which major pathogen is associated with causing the sexually transmitted infection known as syphilis?

a) Neisseria gonorrhoeae

b) Treponema pallidum

c) Chlamydia trachomatis

d) Haemophilus influenzae

7. Which fungal pathogen is responsible for causing athlete’s foot?

a) Candida albicans

b) Aspergillus fumigatus

c) Trichophyton rubrum

d) Cryptococcus neoformans

8. What is the major pathogen associated with the development of dental caries (cavities)?

a) Streptococcus mutans

b) Clostridium difficile

c) Pseudomonas aeruginosa

d) Bacillus anthracis Answer: a) Streptococcus mutans

Page 8 of 92
9. Which virus is considered a major pathogen responsible for causing acquired immunodeficiency
syndrome (AIDS)?

a) Herpes simplex virus

b) Human papillomavirus

c) Human immunodeficiency virus (HIV)

d) Epstein-Barr virus

10. Which major pathogen is responsible for causing the majority of urinary tract infections (UTIs)?

a) Escherichia coli

b) Salmonella enterica

c) Staphylococcus aureus

d) Mycobacterium tuberculosis Answer: a) Escherichia coli

11. Which major pathogen is commonly associated with pneumonia in both adults and children?

a) Streptococcus pyogenes

b) Haemophilus influenzae

c) Pseudomonas aeruginosa

d) Legionella pneumophila

12. What is the major pathogen responsible for causing peptic ulcers in the stomach and duodenum?

a) Streptococcus pneumoniae

b) Helicobacter pylori

c) Escherichia coli

d) Clostridium difficile

13. Which virus is a major pathogen associated with causing cervical cancer?

a) Human papillomavirus (HPV)

b) Influenza virus

c) Rotavirus

d) Measles virus Answer: a) Human papillomavirus (HPV)

Page 9 of 92
14. What is the major pathogen responsible for causing strep throat?

a) Staphylococcus aureus

b) Streptococcus pneumoniae

c) Streptococcus pyogenes

d) Neisseria meningitidis Answer: c) Streptococcus pyogenes

15. Which major pathogen is commonly associated with nosocomial (hospital-acquired) infections,
including urinary tract infections and bloodstream infections?

a) Klebsiella pneumoniae

b) Clostridium difficile

c) Neisseria gonorrhoeae

d) Bordetella pertussis Answer: a) Klebsiella pneumoniae

16. What is the major pathogen responsible for causing the majority of cases of bacterial meningitis in
infants and young children?

a) Listeria monocytogenes

b) Neisseria meningitidis

c) Streptococcus pneumoniae

d) Haemophilus influenzae type b Answer: d) Haemophilus influenzae type b

17. Which major pathogen is associated with causing dental infections and abscesses?

a) Staphylococcus aureus

b) Clostridium botulinum

c) Actinomyces species

d) Enterococcus faecalis Answer: c) Actinomyces species

18. What is the major pathogen responsible for causing the sexually transmitted infection known as
gonorrhea?

a) Treponema pallidum

b) Neisseria gonorrhoeae

c) Chlamydia trachomatis

d) Mycoplasma genitalium Answer: b) Neisseria gonorrhoeae

Page 10 of 92
19. Which major pathogen is associated with causing bacterial food poisoning through the production of a
toxin in contaminated food?

a) Salmonella enterica

b) Vibrio cholerae

c) Clostridium botulinum

d) Staphylococcus aureus

20. What is the major pathogen responsible for causing the majority of cases of dental cavities in humans?

a) Candida albicans

b) Streptococcus mutans

c) Escherichia coli

d) Clostridium difficile Answer: b) Streptococcus mutans

21. Which type of bacteria requires oxygen for growth and metabolism?

a) Anaerobic bacteria

b) Aerobic bacteria

c) Facultative anaerobes

d) Obligate intracellular bacteria

22. What type of bacteria thrives in the presence of oxygen and utilizes it for respiration?

a) Anaerobic bacteria

b) Microaerophilic bacteria

c) Aerobic bacteria

d) Facultative anaerobes Answer: c) Aerobic bacteria

23. Which of the following bacteria is an example of an aerobic pathogen that causes tuberculosis?

a) Clostridium difficile

b) Mycobacterium tuberculosis

c) Escherichia coli

d) Streptococcus pyogenes Answer: b) Mycobacterium tuberculosis

24. Which aerobic bacterium is commonly found in soil and is used for the production of antibiotics like
streptomycin?

Page 11 of 92
a) Staphylococcus aureus

b) Pseudomonas aeruginosa

c) Mycobacterium leprae

d) Streptomyces griseus Answer: d) Streptomyces griseus

25. What is a well-known aerobic bacterium involved in the decomposition of organic matter in the
environment?

a) Clostridium perfringens

b) Bacillus anthracis

c) Escherichia coli

d) Pseudomonas putida Answer: d) Pseudomonas putida

26. Which aerobic bacterium is commonly associated with urinary tract infections?

a) Staphylococcus epidermidis

b) Proteus mirabilis

c) Clostridium difficile

d) Mycoplasma pneumoniae Answer: b) Proteus mirabilis

27. Which aerobic bacterium is responsible for causing whooping cough (pertussis)?

a) Bordetella pertussis

b) Clostridium tetani

c) Salmonella enterica

d) Neisseria meningitidis Answer: a) Bordetella pertussis

28. Which aerobic bacterium is commonly associated with respiratory infections and pneumonia in
immunocompromised individuals?

a) Clostridium botulinum

b) Mycoplasma pneumoniae

c) Streptococcus mutans

d) Escherichia coli Answer: b) Mycoplasma pneumoniae

29. What aerobic bacterium is often found in water and can cause infections in wounds or via ingestion of
contaminated water?

Page 12 of 92
a) Legionella pneumophila

b) Clostridium difficile

c) Listeria monocytogenes

d) Mycobacterium leprae Answer: a) Legionella pneumophila

30. Which aerobic bacterium is associated with the production of greenish-blue pigment and is found in
burn wounds?

a) Staphylococcus aureus

b) Escherichia coli

c) Pseudomonas aeruginosa

d) Clostridium perfringens Answer: c) Pseudomonas aeruginosa

31. What aerobic bacterium is commonly used in the dairy industry to produce yogurt and cheese?

a) Lactobacillus acidophilus

b) Clostridium botulinum

c) Mycobacterium tuberculosis

d) Escherichia coli Answer: a) Lactobacillus acidophilus

32. Which aerobic bacterium is responsible for causing infections in the respiratory tract, particularly in
individuals with cystic fibrosis?

a) Mycobacterium leprae

b) Streptococcus pneumoniae

c) Pseudomonas aeruginosa

d) Clostridium difficile Answer: c) Pseudomonas aeruginosa

33. Which aerobic bacterium is associated with dental plaque formation and gum disease?

a) Clostridium perfringens

b) Streptococcus mutans

c) Listeria monocytogenes

d) Escherichia coli Answer: b) Streptococcus mutans

34. What aerobic bacterium is known for its involvement in causing stomach ulcers?

a) Helicobacter pylori

Page 13 of 92
b) Clostridium difficile

c) Salmonella enterica

d) Mycobacterium tuberculosis Answer: a) Helicobacter pylori

35. Which aerobic bacterium is a common inhabitant of the human skin and is associated with skin
infections?

a) Staphylococcus epidermidis

b) Streptococcus pyogenes

c) Clostridium tetani

d) Mycoplasma pneumoniae Answer: a) Staphylococcus epidermidis

36. What is the primary aerobic bacterium responsible for causing Lyme disease?

a) Treponema pallidum

b) Borrelia burgdorferi

c) Chlamydia trachomatis

d) Neisseria gonorrhoeae Answer: b) Borrelia burgdorferi

37. Which aerobic bacterium is a common cause of urinary tract infections in women?

a) Staphylococcus aureus

b) Clostridium difficile

c) Escherichia coli

d) Mycobacterium leprae Answer: c) Escherichia coli

38. What is a notable aerobic bacterium used in biotechnology for genetic engineering and producing
insulin?

a) Clostridium botulinum

b) Escherichia coli

c) Mycobacterium tuberculosis

d) Streptococcus mutans Answer: b) Escherichia coli

39. Which aerobic bacterium is commonly found in the intestines of animals and can cause foodborne
illnesses in humans?

Page 14 of 92
a) Clostridium perfringens

b) Salmonella enterica

c) Listeria monocytogenes

d) Mycobacterium leprae Answer: b) Salmonella enterica

40. What is the major aerobic bacterium responsible for causing bacterial vaginosis in women?

a) Gardnerella vaginalis

b) Clostridium difficile

c) Streptococcus pneumoniae

d) Neisseria gonorrhoeae Answer: a) Gardnerella vaginalis

Method of disinfection
1. What is the primary goal of sterilization?
a) To reduce the number of microorganisms to a safe level
b) To eliminate all forms of microbial life, including spores
c) To disinfect surfaces and equipment
d) To prevent the growth of bacteria

2. Which method of sterilization involves the use of high-pressure steam to kill microorganisms?
a) Autoclaving
b) Filtration
c) Boiling
d) Pasteurization

3. What is the primary mode of action of ethylene oxide (ETO) as a sterilizing agent?
a) High-temperature steam
b) Alkylating cellular components
c) Oxidizing cellular proteins
d) Filtration of microorganisms

4. Which of the following is a chemical disinfectant commonly used for disinfection of surfaces in
healthcare settings?
a) Filtration
b) Boiling
c) Hydrogen peroxide
d) Autoclaving

5. What is the primary advantage of using ultraviolet (UV) radiation for disinfection?
a) It is effective in killing bacterial spores
b) It is non-toxic to humans
c) It requires high temperatures
d) It is ineffective against viruses

Page 15 of 92
6. Which method of disinfection involves the removal of microorganisms by passing a liquid or gas
through a filter with tiny pores?
a) Autoclaving
b) Pasteurization
c) Filtration
d) Ethylene oxide sterilization

7. What is the purpose of using chemical disinfectants like chlorine and iodine?
a) To eliminate all forms of microbial life
b) To reduce the number of microorganisms to a safe level
c) To remove dirt and debris from surfaces
d) To enhance the growth of beneficial microorganisms

8. Which method of disinfection is commonly used for disinfecting drinking water and swimming pool
water?
a) Filtration
b) Ultraviolet (UV) radiation
c) Chlorination
d) Autoclaving

9. Which of the following methods of sterilization is NOT suitable for heat-sensitive materials?
a) Autoclaving
b) Dry heat sterilization
c) Ethylene oxide (ETO) sterilization
d) Gamma radiation

10. What is the primary advantage of using gamma radiation for sterilization?
a) It is inexpensive
b) It can be used for heat-sensitive materials
c) It requires minimal exposure time
d) It leaves a residue on sterilized items

11. Which chemical disinfectant is often used for skin antisepsis before medical procedures?
a) Iodine
b) Bleach (sodium hypochlorite)
c) Hydrogen peroxide
d) Ethanol

12. What is the primary mechanism of action of hydrogen peroxide as a disinfectant?


a) Alkylating cellular components
b) Oxidizing cellular proteins and enzymes
c) Heating microbial cells
d) Removing microorganisms by filtration

13. Which of the following is a limitation of using UV-C radiation for disinfection?
a) It is effective against bacterial spores
b) It requires minimal contact time
c) It is ineffective in air purification
d) It does not penetrate surfaces or materials

Page 16 of 92
14. What is the primary purpose of pasteurization in the food industry?
a) To sterilize food products
b) To extend the shelf life of food
c) To improve the taste of food
d) To remove all microorganisms from food

15. Which sterilization method relies on the use of high-frequency sound waves to disrupt and kill
microorganisms?
a) Autoclaving
b) Ultrasonication
c) Filtration
d) Dry heat sterilization

16. What is the primary purpose of filtration as a method of disinfection?


a) To eliminate bacterial spores
b) To remove dirt and debris from liquids
c) To expose microorganisms to high temperatures
d) To irradiate microorganisms with UV light Answer: b) To remove dirt and debris from liquids

17. Which of the following is a drawback of using ethylene oxide (ETO) for sterilization?
a) It is highly flammable
b) It leaves a residue on sterilized items
c) It requires high temperatures
d) It is ineffective against bacteria Answer: a) It is highly flammable

18. Which method of disinfection involves the use of chemicals or detergents to remove dirt and organic
material from surfaces before applying a disinfectant?
a) Dry heat sterilization
b) Precleaning
c) Ethylene oxide (ETO) sterilization
d) Gamma radiation Answer: b) Precleaning

19. What is the primary limitation of using dry heat sterilization compared to moist heat methods like
autoclaving?
a) It requires longer exposure times
b) It cannot penetrate materials effectively
c) It is less energy-efficient
d) It is not suitable for heat-sensitive materials Answer: b) It cannot penetrate materials effectively

20. Which disinfection method is often used for decontaminating laboratory equipment like pipettes and
glassware?
a) Autoclaving
b) Pasteurization
c) Chemical disinfection
d) Ultraviolet (UV) radiation Answer: c) Chemical disinfection

21. Which method of disinfection primarily relies on the use of high temperatures to kill or inactivate
microorganisms?
a) Chemical disinfection
b) Physical disinfection

Page 17 of 92
c) Ultraviolet (UV) radiation
d) Filtration Answer: b) Physical disinfection

22. Which of the following is NOT a commonly used chemical disinfectant in microbiology?
a) Chlorine
b) Ethanol
c) Hydrogen peroxide
d) Glassware Answer: d) Glassware

23. What is the primary mechanism of action of chemical disinfectants like chlorine and iodine?
a) Mechanical removal of microorganisms
b) Altering the pH of the environment
c) Disrupting the cell membrane or proteins
d) Providing nutrients for microorganisms Answer: c) Disrupting the cell membrane or proteins

24. Which of the following is a method of disinfection commonly used in water treatment to kill or
inactivate microorganisms?
a) Autoclaving
b) Pasteurization
c) Filtration
d) Incineration Answer: b) Pasteurization

25. Ultraviolet (UV) radiation is effective in disinfection primarily by:


a) Causing mutations in microbial DNA
b) Breaking down cell walls
c) Heating the microbial cells
d) Providing nutrients to the microorganisms Answer: a) Causing mutations in microbial DNA

Bacterial Vaccine
1. What is the primary purpose of a bacterial vaccine?
A. To treat bacterial infections
B. To prevent bacterial infections
C. To boost the immune system
D. To cure viral diseases

2. Which of the following is NOT a type of bacterial vaccine?


A. Live attenuated vaccine
B. Inactivated (killed) vaccine
C. Subunit vaccine
D. Antibiotic vaccine

3. Which bacterial vaccine type typically provides long-lasting immunity with a single dose?
A. Live attenuated vaccine
B. Inactivated (killed) vaccine
C. Toxoid vaccine
D. Subunit vaccine

4. Which bacterial vaccine type involves using only specific components of the bacterial
cell, such as proteins or polysaccharides?

Page 18 of 92
A. Live attenuated vaccine
B. Inactivated (killed) vaccine
C. Toxoid vaccine
D. Subunit vaccine

5. What is a toxoid vaccine primarily designed to target?


A. Bacterial cell walls
B. Bacterial toxins
C. Bacterial DNA
D. Bacterial flagella

6. Which bacterial vaccine type is safer for individuals with weakened immune systems?
A. Live attenuated vaccine
B. Inactivated (killed) vaccine
C. Toxoid vaccine
D. Subunit vaccine

7. Which bacterial vaccine is used to prevent diseases like diphtheria and tetanus?
A. Live attenuated vaccine
B. Inactivated (killed) vaccine
C. Toxoid vaccine
D. Subunit vaccine

8. Which bacterial vaccine type can provide protection against multiple strains or serotypes
of the same bacterium?
A. Live attenuated vaccine
B. Inactivated (killed) vaccine
C. Toxoid vaccine
D. Subunit vaccine

9. Which bacterial vaccine type often requires booster shots to maintain immunity over
time?
A. Live attenuated vaccine
B. Inactivated (killed) vaccine
C. Toxoid vaccine
D. Subunit vaccine

10. What is the main advantage of bacterial vaccines over antibiotics in controlling bacterial
infections?
A. Bacterial vaccines work faster
B. Bacterial vaccines are more effective
C. Bacterial vaccines prevent infections
D. Bacterial vaccines have fewer side effects

11. Which of the following is an example of a live attenuated bacterial vaccine?


A. Inactivated tuberculosis vaccine (BCG)
B. Tetanus toxoid vaccine
C. Inactivated polio vaccine (IPV)
D. Oral cholera vaccine

Page 19 of 92
12. What is the primary function of an adjuvant in a bacterial vaccine?
A. To weaken the bacteria
B. To enhance the vaccine’s shelf life
C. To stimulate the immune response
D. To replace the need for boosters

13. Which bacterial vaccine is commonly administered to travelers going to regions with a
high risk of exposure to Vibrio cholerae?
A. Typhoid vaccine
B. Shigella vaccine
C. Cholera vaccine
D. Salmonella vaccine

14. Which bacterial vaccine type is used to prevent infections caused by Neisseria
meningitidis?
A. Live attenuated vaccine
B. Inactivated (killed) vaccine
C. Toxoid vaccine
D. Conjugate vaccine

15. What is the purpose of a “booster” dose in a bacterial vaccine schedule?


A. To reduce the side effects of the vaccine
B. To increase the vaccine’s shelf life
C. To remind the immune system to maintain immunity
D. To replace the initial vaccine dose

16. Which bacterial vaccine is used to prevent diseases like whooping cough and
pertussis?
A. Tuberculosis vaccine
B. Tetanus toxoid vaccine
C. Diphtheria-pertussis-tetanus (DPT) vaccine
D. Haemophilus influenzae type b (Hib) vaccine

17. Which bacterial vaccine is given to prevent Streptococcus pneumoniae infections in


young children?
A. Hib vaccine
B. Pneumococcal conjugate vaccine (PCV)
C. Meningococcal vaccine
D. DPT vaccine

18. Which bacterial vaccine is used to prevent Lyme disease?


A. Tuberculosis vaccine
B. Tetanus toxoid vaccine
C. Lyme disease vaccine
D. Influenza vaccine

19. What is the primary target of the human papillomavirus (HPV) vaccine?
A. Bacterial infections
B. Viral infections

Page 20 of 92
C. Fungal infections
D. Parasitic infections

20. Which bacterial vaccine type is made from bacterial cell fragments, such as the cell
wall?
A. Live attenuated vaccine
B. Inactivated (killed) vaccine
C. Toxoid vaccine
D. Subunit vaccine Answer: D. Subunit vaccine

21. Which bacterial vaccine is used to prevent tuberculosis?


A. Pneumococcal vaccine
B. Inactivated polio vaccine (IPV)
C. Bacillus Calmette-Guérin (BCG) vaccine
D. Tetanus toxoid vaccine Answer: C. Bacillus Calmette-Guérin (BCG) vaccine

22. Which bacterial vaccine is used to prevent infections caused by Clostridium tetani?
A. Pertussis vaccine
B. Tuberculosis vaccine
C. Typhoid vaccine
D. Tetanus toxoid vaccine

23. Which bacterial vaccine type is used to prevent infections caused by Haemophilus
influenzae type b (Hib)?
A. Conjugate vaccine
B. Inactivated (killed) vaccine
C. Toxoid vaccine
D. Subunit vaccine

24. Which bacterial vaccine is given to protect against bacterial pneumonia and ear
infections in young children?
A. Influenza vaccine
B. Pneumococcal conjugate vaccine (PCV)
C. Hepatitis B vaccine
D. Diphtheria-pertussis-tetanus (DPT) vaccine Answer: B. Pneumococcal conjugate
vaccine (PCV)

25. Which bacterial vaccine is used to prevent diseases caused by the bacterium
Bordetella pertussis?
A. Tuberculosis vaccine
B. Tetanus toxoid vaccine
C. Pertussis vaccine
D. Meningococcal vaccine Answer: C. Pertussis vaccine

26. Which bacterial vaccine is given to prevent infections caused by Salmonella typhi, the
bacterium responsible for typhoid fever?
A. Typhoid vaccine
B. Cholera vaccine

Page 21 of 92
C. Diphtheria-pertussis-tetanus (DPT) vaccine
D. Tuberculosis vaccine Answer: A. Typhoid vaccine

27. Which bacterial vaccine type contains a weakened but live version of the bacterium?
A. Inactivated (killed) vaccine
B. Conjugate vaccine
C. Live attenuated vaccine
D. Subunit vaccine Answer: C. Live attenuated vaccine

28. Which bacterial vaccine is used to prevent infections caused by the bacterium that can
lead to cervical cancer?
A. Tuberculosis vaccine
B. Tetanus toxoid vaccine
C. Hepatitis B vaccine
D. HPV vaccine Answer: D. HPV vaccine

29. Which bacterial vaccine is used to prevent diseases like pneumonia, meningitis, and
septicemia caused by Streptococcus pneumoniae?
A. Pneumococcal conjugate vaccine (PCV)
B. Tetanus toxoid vaccine
C. Tuberculosis vaccine
D. Pertussis vaccine Answer: A. Pneumococcal conjugate vaccine (PCV)

30. Which bacterial vaccine is administered to prevent infections caused by the bacterium
responsible for causing anthrax?
A. Anthrax vaccine
B. Tuberculosis vaccine
C. Meningococcal vaccine
D. Cholera vaccine Answer: A. Anthrax vaccine

Antimicrobial drugs resistance


1. What is the primary mechanism by which bacteria develop resistance to antibiotics?
a) Mutation
b) Conjugation
c) Transformation
d) All of the above
Answer: b) Conjugation

2. Which of the following statements about antibiotic resistance is true?


a) Antibiotic resistance is only a problem in developing countries.
b) Antibiotic resistance is solely caused by the overuse of antibiotics.
c) Antibiotic resistance can develop naturally over time.
d) Antibiotic resistance is reversible once it occurs.
Answer: c) Antibiotic resistance can develop naturally over time.

3. What term describes the ability of bacteria to survive exposure to an antibiotic that would normally kill
them?
a) Antibiotic susceptibility

Page 22 of 92
b) Antibiotic tolerance
c) Antibiotic resistance
d) Antibiotic persistence
Answer: c) Antibiotic resistance

4. Which of the following is NOT a common way that bacteria acquire antibiotic-resistance genes?
a) Mutation
b) Horizontal gene transfer
c) Transformation
d) Vertical gene transfer
Answer: d) Vertical gene transfer

5. How can antibiotic resistance be slowed down or minimized?


a) Prescribing antibiotics for viral infections
b) Using broad-spectrum antibiotics whenever possible
c) Completing the full course of antibiotics as prescribed
d) Using antibiotics only when absolutely necessary
Answer: d) Using antibiotics only when absolutely necessary

6. Which type of antibiotic is most susceptible to resistance development due to its narrow spectrum of
activity?
a) Broad-spectrum antibiotics
b) Penicillins
c) Macrolides
d) Fluoroquinolones
Answer: b) Penicillins

7. What is one of the major consequences of antibiotic resistance?


a) Decreased healthcare costs
b) Increased treatment success rates
c) Longer hospital stays
d) Lower patient mortality rates
Answer: c) Longer hospital stays

8. What is the primary target of antibiotics in bacterial cells?


a) Ribosomes
b) Cell membrane
c) Nucleus
d) Mitochondria
Answer: a) Ribosomes

9. Which of the following best describes a superbug?


a) A term used to describe a particularly large bacterium
b) A bacterium that has developed resistance to multiple antibiotics
c) A bacterium that is sensitive to all antibiotics
d) A bacterium that is found in extreme environments
Answer: b) A bacterium that has developed resistance to multiple antibiotics

10. What is one of the strategies to combat antibiotic resistance?


a) Prescribing higher doses of antibiotics

Page 23 of 92
b) Developing new antibiotics more slowly
c) Using combination therapy with multiple antibiotics
d) Discontinuing the use of antibiotics altogether
Answer: c) Using combination therapy with multiple antibiotics

11. Which of the following is an example of a gram-negative bacterium known for its antibiotic resistance?
a) Staphylococcus aureus
b) Escherichia coli (E. coli)
c) Streptococcus pyogenes
d) Clostridium difficile
Answer: b) Escherichia coli (E. coli)

12. What is the primary reason behind the overuse and misuse of antibiotics?
a) To promote the development of antibiotic resistance
b) To reduce the cost of healthcare
c) To prevent bacterial infections
d) To speed up the recovery from viral infections
Answer: d) To speed up the recovery from viral infections

13. What is the term for the phenomenon where bacteria that were once susceptible to an antibiotic
become resistant to it over time?
a) Antibiotic mutation
b) Antibiotic insensitivity
c) Antibiotic switching
d) Antibiotic adaptation
Answer: d) Antibiotic adaptation

14. Which of the following factors contributes to the rapid spread of antibiotic resistance in healthcare
settings?
a) Strict infection control measures
b) Rapid development of new antibiotics
c) Frequent patient transfers between healthcare facilities
d) Limited use of antibiotics
Answer: c) Frequent patient transfers between healthcare facilities

15. What is the name of the enzyme produced by some bacteria that inactivates beta-lactam antibiotics like
penicillin?
a) Ribonuclease
b) Beta-lactamase
c) DNA polymerase
d) RNA ligase
Answer: b) Beta-lactamase

16. Which of the following is NOT a common mechanism of antibiotic resistance in bacteria?
a) Efflux pumps
b) Cell wall synthesis inhibition
c) Target site modification
d) Enhanced antibiotic uptake
Answer: d) Enhanced antibiotic uptake

Page 24 of 92
17. What is the term for the process by which antibiotic-resistant genes can be transferred from one
bacterial species to another through direct contact or exchange of genetic material?
a) Vertical gene transfer
b) Horizontal gene transfer
c) Gene inversion
d) Gene duplication
Answer: b) Horizontal gene transfer

18. Which of the following infections is most likely to be caused by a superbug?


a) Common cold
b) Strep throat
c) Urinary tract infection
d) Tuberculosis
Answer: d) Tuberculosis

19. What can individuals do to help combat antibiotic resistance?


a) Request antibiotics for viral infections
b) Share antibiotics with friends and family
c) Follow healthcare provider’s instructions for antibiotic use
d) Store antibiotics in a warm, humid environment
Answer: c) Follow the healthcare provider’s instructions for antibiotic use

20. Which class of antibiotics is known for its potential to cause Clostridium difficile infection, a common
healthcare-associated infection?
a) Tetracyclines
b) Macrolides
c) Fluoroquinolones
d) Aminoglycosides
Answer: c) Fluoroquinolones

21. What is the primary mode of action of antibiotics that inhibit bacterial protein synthesis, such as
tetracyclines?
a) Disrupting cell membrane integrity
b) Inhibiting DNA replication
c) Inhibiting RNA synthesis
d) Blocking ribosome function
Answer: d) Blocking ribosome function

22. Which of the following best describes “antibiotic stewardship”?


a) The practice of prescribing antibiotics for every suspected bacterial infection
b) The responsible use of antibiotics to preserve their effectiveness and minimize resistance
c) The rapid development of new antibiotics to combat resistance
d) The promotion of over-the-counter antibiotic sales
Answer: b) The responsible use of antibiotics to preserve their effectiveness and minimize resistance

23. Which of the following is an example of an antibiotic-resistant “superbug” that has caused significant
healthcare challenges?
a) Streptococcus pneumoniae
b) Mycobacterium tuberculosis
c) Methicillin-resistant Staphylococcus aureus (MRSA)

Page 25 of 92
d) Salmonella enterica
Answer: c) Methicillin-resistant Staphylococcus aureus (MRSA)

24. What is the term for the phenomenon where antibiotics that are no longer effective against a bacterial
infection must be replaced by more potent antibiotics?
a) Antibiotic escalation
b) Antibiotic cycling
c) Antibiotic rotation
d) Antibiotic substitution
Answer: b) Antibiotic cycling

25. Which of the following is NOT a common strategy used by bacteria to resist the action of antibiotics?
a) Increasing antibiotic uptake
b) Modifying antibiotic targets
c) Inactivating antibiotics
d) Decreasing antibiotic efflux
Answer: a) Increasing antibiotic uptake

26. What is the primary reason behind the development of antibiotic resistance in agriculture?
a) The use of antibiotics to treat sick animals
b) The use of antibiotics as growth promoters
c) The absence of antibiotics in animal agriculture
d) The implementation of strict regulations on antibiotic use
Answer: b) The use of antibiotics as growth promoters

27. Which of the following is a key strategy to prevent the spread of antibiotic resistance within healthcare
facilities?
a) Frequent use of broad-spectrum antibiotics
b) Strict hand hygiene and infection control measures
c) Prescribing antibiotics for viral infections
d) Using antibiotics without a prescription
Answer: b) Strict hand hygiene and infection control measures

28. What is the term for a bacterium that is resistant to all known antibiotics and poses a significant threat
to public health?
a) Antibiotic-sensitive bacterium
b) Antibiotic-tolerant bacterium
c) Extensively drug-resistant bacterium
d) Antibiotic-resistant bacterium
Answer: c) Extensively drug-resistant bacterium

29. Which class of antibiotics is known for its activity against Mycobacterium tuberculosis, the bacterium
that causes tuberculosis (TB)?
a) Beta-lactams
b) Aminoglycosides
c) Quinolones
d) Rifamycins
Answer: d) Rifamycins

Page 26 of 92
30). How can plasmids contribute to drug resistance in bacteria?
a) By encoding antibiotic drugs
b) By providing a physical barrier to antibiotics
c) By carrying genes that confer resistance to antibiotics
d) By blocking antibiotic entry into bacterial cells
Answer: c) By carrying genes that confer resistance to antibiotics

31. How can genetic mutations contribute to antibiotic resistance in bacteria?


a) They make bacteria more susceptible to antibiotics.
b) They reduce bacterial replication rates.
c) They can alter the structure of antibiotic targets, making them less susceptible to antibiotics.
d) They increase bacterial sensitivity to antibiotics.
Answer: c) They can alter the structure of antibiotic targets, making them less susceptible to antibiotics.

32. Which of the following is an example of a plasmid-encoded resistance gene often found in bacteria?
a) Tetracyclinas
b) Penicillinase
c) Streptomycinase
d) Ampicillinase

33. Plasmid-mediated drug resistance can spread between bacteria of different species through a process
called:
a) Binary fission
b) Conjugation

34. Which of the following antibiotics is often affected by plasmid-mediated drug resistance mechanisms?
a) Antivirals
b) Antifungals
c) Beta-lactam antibiotics

35. What is the primary advantage of plasmid-mediated drug resistance for bacteria?
a) Increased susceptibility to antibiotics
b) Enhanced growth rate
c) Improved DNA replication
d) Survival in the presence of antibiotics

Microbiology
1. What is the primary mechanism of action of antibiotics that target bacterial cell walls?
A) Inhibition of DNA replication
B) Disruption of cell membrane integrity
C) Inhibition of protein synthesis
D) Inhibition of cell wall synthesis
Answer: D) Inhibition of cell wall synthesis

2. Which of the following antibiotics causes red man syndrome?


A) Vancomycin
B) Ciprofloxacin

Page 27 of 92
3. Which of the following antibiotics interferes with protein synthesis by binding to the 30S ribosomal subunit
in bacteria?
A) Penicillin
B) Ciprofloxacin
C) Tetracycline
D) Vancomycin
Answer: C) Tetracycline

4. Which of the following antibiotics cause PHOTOTOXIC effects?


A) Penicillin
B) Sulphamethoxazole

5. How do sulfonamide antibiotics work to inhibit bacterial growth?


A) Inhibition of DNA gyrase
B) Inhibition of cell wall synthesis
C) Competitive inhibition of folic acid synthesis
D) Disruption of the cell membrane
Answer: C) Competitive inhibition of folic acid synthesis

6. Which class of antimicrobial drugs targets fungal infections by disrupting the ergosterol component of
fungal cell membranes?
A) Aminoglycosides
B) Azoles
C) Cephalosporins
D) Quinolones
Answer: B) Azoles

7. What is the primary mechanism of action of antiviral drugs that target reverse transcriptase in
retroviruses?
A) Inhibition of viral entry
B) Inhibition of viral fusion
C) Inhibition of viral DNA replication
D) Inhibition of viral RNA synthesis
Answer: C) Inhibition of viral DNA replication

8. Which antimicrobial drug is commonly used to treat tuberculosis (TB) by inhibiting the synthesis of
mycobacterial cell wall components?
A) Ampicillin
B) Rifampin
C) Gentamicin
D) Fluconazole
Answer: B) Rifampin

9. What is the primary mechanism of action of antifungal drugs that target the synthesis of beta-glucans in
fungal cell walls?
A) Inhibition of RNA polymerase
B) Inhibition of ergosterol synthesis
C) Inhibition of beta-glucan synthesis
D) Inhibition of protein synthesis
Answer: C) Inhibition of beta-glucan synthesis

Page 28 of 92
10. Which of the following antibiotics disrupts bacterial cell membranes by creating channels or pores in the
lipid bilayer?
A) Penicillin
B) Ciprofloxacin
C) Polymyxin
D) Tetracycline
Answer: C) Polymyxin

11. How do antiprotozoal drugs like metronidazole work against parasitic infections?
A) Inhibition of DNA replication
B) Inhibition of cell wall synthesis
C) Inhibition of protein synthesis
D) Inhibition of RNA synthesis
Answer: A) Inhibition of DNA replication

12. Which antimicrobial drug class is known for inhibiting the activity of topoisomerases in bacteria,
preventing DNA supercoiling?
A) Aminoglycosides
B) Macrolides
C) Quinolones
D) Sulfonamides
Answer: C) Quinolones

13. Which antiviral drug inhibits the action of the viral enzyme protease, preventing the cleavage of viral
polyproteins into functional proteins?
A) Zanamivir
B) Oseltamivir
C) Ritonavir
D) Acyclovir
Answer: C) Ritonavir

14. What is the primary mechanism of action of aminoglycoside antibiotics such as gentamicin?
A) Inhibition of RNA polymerase
B) Inhibition of cell wall synthesis
C) Inhibition of protein synthesis
D) Inhibition of DNA replication
Answer: C) Inhibition of protein synthesis

15. Which antimicrobial drug class targets the 50S ribosomal subunit in bacteria, interfering with protein
synthesis?
A) Macrolides
B) Penicillins
C) Fluoroquinolones
D) Sulfonamides
Answer: A) Macrolides

16. How do antiviral drugs like acyclovir work against herpes simplex viruses (HSV)?
A) Inhibition of viral entry
B) Inhibition of viral fusion
C) Inhibition of viral DNA polymerase

Page 29 of 92
D) Inhibition of viral reverse transcriptase
Answer: C) Inhibition of viral DNA polymerase

17. Which antimicrobial drug is effective against both gram-positive and gram-negative bacteria and disrupts
bacterial cell membranes?
A) Chloramphenicol
B) Vancomycin
C) Polymyxin
D) Cephalosporin
Answer: C) Polymyxin

18. What is the primary mechanism of action of antifungal drugs known as echinocandins?
A) Inhibition of DNA replication
B) Inhibition of ergosterol synthesis
C) Inhibition of chitin synthesis
D) Inhibition of protein synthesis
Answer: C) Inhibition of chitin synthesis

19. Which antiparasitic drug disrupts the mitochondrial electron transport chain in parasites, leading to their
death?
A) Mebendazole
B) Artemisinin
C) Atovaquone
D) Ivermectin
Answer: C) Atovaquone

20. What is the primary mechanism of action of penicillin and other beta-lactam antibiotics?
A) Inhibition of DNA replication
B) Inhibition of RNA synthesis
C) Inhibition of protein synthesis
D) Inhibition of cell wall synthesis
Answer: D) Inhibition of cell wall synthesis

21. Which antiviral drug is commonly used to treat HIV by inhibiting the reverse transcriptase enzyme?
A) Ribavirin
B) Zidovudine (AZT)
C) Amantadine
D) Griseofulvin
Answer: B) Zidovudine (AZT)

22. How do antifungal drugs like amphotericin B work to kill fungal cells?
A) Inhibition of ergosterol synthesis
B) Inhibition of cell wall synthesis
C) Inhibition of RNA polymerase
D) Inhibition of DNA replication
Answer: A) Inhibition of ergosterol synthesis

23. Which antibiotic class is known for disrupting bacterial protein synthesis by binding to the 50S ribosomal
subunit?
A) Aminoglycosides

Page 30 of 92
B) Tetracyclines
C) Penicillins
D) Quinolones
Answer: B) Tetracyclines

24. What is the primary mechanism of action of metronidazole, an antibiotic used to treat anaerobic bacterial
infections and protozoal infections?
A) Inhibition of DNA gyrase
B) Inhibition of cell wall synthesis
C) Inhibition of DNA replication
D) Inhibition of RNA synthesis
Answer: C) Inhibition of DNA replication

25. Which antiviral drug targets the viral neuraminidase enzyme, preventing the release of newly formed
virions from infected cells?
A) Ritonavir
B) Oseltamivir
C) Acyclovir
D) Ribavirin
Answer: B) Oseltamivir

26. How do antimalarial drugs like chloroquine work to combat Plasmodium parasites?
A) Inhibition of protein synthesis
B) Inhibition of DNA replication
C) Inhibition of heme detoxification
D) Inhibition of cell wall synthesis
Answer: C) Inhibition of heme detoxification

27. Which class of antimicrobial drugs includes drugs like isoniazid and ethambutol used in the treatment of
tuberculosis (TB)?
A) Aminoglycosides
B) Quinolones
C) Antifolates
D) Antituberculosis drugs
Answer: D) Antituberculosis drugs

28. What is the primary mechanism of action of daptomycin, an antibiotic used to treat gram-positive bacterial
infections?
A) Inhibition of DNA replication
B) Inhibition of RNA synthesis
C) Disruption of the cell membrane
D) Inhibition of protein synthesis
Answer: C) Disruption of the cell membrane

29. Which antiparasitic drug is used to treat head lice and scabies by interfering with nerve transmission in
parasites?
A) Mebendazole
B) Pyrantel pamoate
C) Permethrin

Page 31 of 92
D) Praziquantel
Answer: C) Permethrin

30. What is the primary mechanism of action of mupirocin, an antibiotic used for skin infections?
A) Inhibition of RNA synthesis
B) Inhibition of DNA replication
C) Inhibition of protein synthesis
D) Inhibition of cell wall synthesis
Answer: D) Inhibition of cell wall synthesis

31. Which antiviral drug inhibits the activity of the HIV enzyme integrase, preventing the integration of viral
DNA into the host genome?
A) Zidovudine (AZT)
B) Raltegravir
C) Lamivudine
D) Ribavirin
Answer: B) Raltegravir

32. How do antifungal drugs like griseofulvin work against dermatophyte infections?
A) Inhibition of DNA replication
B) Inhibition of RNA synthesis
C) Inhibition of ergosterol synthesis
D) Inhibition of protein synthesis
Answer: A) Inhibition of DNA replication

33. Certainly! Here are some additional multiple-choice questions (MCQs) on antimicrobial drugs and their
mechanisms of action:

34. Which antibiotic class includes drugs like cephalexin and ceftriaxone, known for their broad-spectrum
activity against both gram-positive and gram-negative bacteria?
A) Penicillins
B) Macrolides
C) Cephalosporins
D) Sulfonamides
Answer: C) Cephalosporins

35. What is the primary mechanism of action of polymyxin B, an antibiotic used as a last-resort treatment for
multidrug-resistant gram-negative bacteria?
A) Inhibition of DNA replication
B) Inhibition of RNA synthesis
C) Disruption of the cell membrane
D) Inhibition of protein synthesis
Answer: C) Disruption of the cell membrane

36. Which antiviral drug interferes with the viral RNA-dependent RNA polymerase, inhibiting the replication of
RNA viruses like hepatitis C?
A) Ribavirin
B) Zanamivir
C) Sofosbuvir

Page 32 of 92
D) Nevirapine
Answer: C) Sofosbuvir

37. How do antiprotozoal drugs like chloroquine work against protozoal parasites like Plasmodium species?
A) Inhibition of DNA replication
B) Inhibition of protein synthesis
C) Inhibition of cell wall synthesis
D) Inhibition of RNA synthesis
Answer: A) Inhibition of DNA replication

38. Which class of antimicrobial drugs is primarily used to treat urinary tract infections and works by inhibiting
bacterial dihydrofolate reductase?
A) Penicillins
B) Tetracyclines
C) Sulfonamides
D) Aminoglycosides
Answer: C) Sulfonamides

39. What is the primary mechanism of action of nitrofurantoin, an antibiotic commonly used for urinary tract
infections?
A) Inhibition of DNA replication
B) Inhibition of cell wall synthesis
C) Disruption of the cell membrane
D) Inhibition of protein synthesis
Answer: A) Inhibition of DNA replication

40. Which antiviral drug targets the viral protease enzyme and is used to treat hepatitis C infections?
A) Oseltamivir
B) Ritonavir
C) Ledipasvir
D) Ribavirin
Answer: C) Ledipasvir

41. How do antifungal drugs like fluconazole work to treat fungal infections?
A) Inhibition of ergosterol synthesis
B) Inhibition of RNA synthesis
C) Inhibition of chitin synthesis
D) Inhibition of protein synthesis
Answer: A) Inhibition of ergosterol synthesis

42. Which antimicrobial drug is used to treat leprosy (Hansen’s disease) by inhibiting the synthesis of mycolic
acids in Mycobacterium species?
A) Rifampin
B) Streptomycin
C) Dapsone
D) Isoniazid
Answer: D) Isoniazid

43. What is the primary mechanism of action of nystatin, an antifungal drug used to treat candidiasis and
other fungal infections?

Page 33 of 92
A) Inhibition of ergosterol synthesis
B) Inhibition of RNA synthesis
C) Inhibition of chitin synthesis
D) Inhibition of DNA replication
Answer: A) Inhibition of ergosterol synthesis

Laboratory Diagnosis
1. Which staining technique is commonly used to differentiate bacterial species into Gram-positive and
Gram-negative groups?
a) Acid-fast staining
b) Capsule staining
c) Flagella staining
d) Gram staining Answer: d) Gram staining

2. The main purpose of throat culture is to detect the.


a) Staph Aureus
b) Strept pyogenes

3. What is the primary goal of PCR (Polymerase Chain Reaction) in microbiology diagnosis?
a) Identifying bacterial morphology
b) Amplifying and detecting specific DNA sequences
c) Measuring antibiotic susceptibility
d) Determining serological reactions Answer: b) Amplifying and detecting specific DNA sequences

4. Serological tests are used to detect:


a) Bacterial endospores
b) Viral RNA
c) Antibodies or antigens in a patient’s blood serum
d) Fungal cell walls Answer: c) Antibodies or antigens in a patient’s blood serum

5. What is the main purpose of an antibiotic susceptibility test?


a) Identifying bacterial species
b) Determining the cause of an infection
c) Assessing the effectiveness of antibiotics against a specific microorganism
d) Detecting viral pathogens Answer: c) Assessing the effectiveness of antibiotics against a specific
microorganism

6. If the venipuncture site is inadequately disinfected, a blood culture bottle is most often contaminated with
which kind of bacteria?
a) E.coli
b) Staph epidermidis

7. Which laboratory technique allows for the identification of microorganisms based on their genetic
material?
a) Gram staining
b) PCR (Polymerase Chain Reaction)
c) Serological testing
d) MALDI-TOF mass spectrometry Answer: b) PCR (Polymerase Chain Reaction)

Page 34 of 92
8. What is the primary purpose of MALDI-TOF mass spectrometry in microbiology?
a) Amplifying DNA sequences
b) Identifying fungal spores
c) Analyzing protein profiles for bacterial and fungal identification
d) Determining bacterial motility Answer: c) Analyzing protein profiles for bacterial and fungal
identification

9. Culture-independent diagnostic methods in microbiology are used primarily for:


a) Identifying bacteria based on colony morphology
b) Rapid growth of bacteria in liquid media
c) Detecting microorganisms without the need for culturing
d) Assessing antibiotic resistance Answer: c) Detecting microorganisms without the need for culturing

10. In a microbiology laboratory, what is the primary purpose of using biosafety cabinets?
a) Sterilizing laboratory equipment
b) Ensuring laboratory personnel wear PPE
c) Providing a controlled environment for handling microorganisms
d) Amplifying DNA sequences Answer: c) Providing a controlled environment for handling
microorganisms

11. Which of the following is a safety measure to protect against biohazardous materials in a microbiology
laboratory?
a) Wearing lab coats
b) Using glassware without gloves
c) Working in an open laboratory environment
d) Disposing of hazardous materials in regular waste bins Answer: a) Wearing lab coats

12. Which of the following techniques involves the cultivation of microorganisms on selective media to
encourage the growth of specific species?
a) Streak plating
b) Enrichment culturing
c) Gram staining
d) Acid-fast staining Answer: b) Enrichment culturing

13. In microbiology, the term “metagenomics” refers to:


a) Studying the metabolism of microorganisms
b) Identifying viruses using genetic markers
c) Analyzing the collective genetic material of microbial communities
d) Measuring the sensitivity of microorganisms to antibiotics Answer: c) Analyzing the collective genetic
material of microbial communities

14. Which of the following is NOT a common application of molecular diagnostics in microbiology?
a) Identifying the genetic basis of antibiotic resistance
b) Detecting viral RNA in blood samples
c) Analyzing bacterial colony morphology
d) Investigating the genetic diversity of microbial populations Answer: c) Analyzing bacterial colony
morphology

15. What is the primary purpose of the Kirby-Bauer disk diffusion method in microbiology?
a) Identifying bacterial species

Page 35 of 92
b) Measuring bacterial motility
c) Assessing antibiotic susceptibility
d) Detecting fungal infections Answer: c) Assessing antibiotic susceptibility

16. Which of the following laboratory techniques is used to visualize the structural components of viruses?
a) Gram staining
b) Electron microscopy
c) MALDI-TOF mass spectrometry
d) PCR (Polymerase Chain Reaction) Answer: b) Electron microscopy

17. What type of microscope is commonly used for examining live microorganisms in a microbiology
laboratory?
a) Scanning electron microscope (SEM)
b) Transmission electron microscope (TEM)
c) Phase-contrast microscope
d) Confocal microscope Answer: c) Phase-contrast microscope

18. In a microbiology laboratory, what does the term “aseptic technique” refer to?
a) The use of antimicrobial agents to sterilize equipment
b) Proper handling and manipulation of microorganisms to prevent contamination
c) The use of antibiotics to inhibit bacterial growth
d) Rapid culture growth under high-pressure conditions Answer: b) Proper handling and manipulation of
microorganisms to prevent contamination

19. Which of the following is NOT a category of personal protective equipment (PPE) used in microbiology
laboratories?
a) Lab coats
b) Safety goggles
c) Hairnets
d) Office attire Answer: d) Office attire

20. What is the primary advantage of using selective media in microbiology?


a) Promoting the growth of all microorganisms equally
b) Allowing the growth of only specific microorganisms
c) Enhancing the visibility of microbial colonies
d) Increasing the rate of antibiotic resistance Answer: b) Allowing the growth of only specific
microorganisms

21. What is the main purpose of a biosafety level (BSL) designation in a microbiology laboratory?
a) To indicate the level of cleanliness in the laboratory
b) To specify the types of microorganisms that can be handled safely
c) To determine the color of laboratory equipment
d) To regulate laboratory funding Answer: b) To specify the types of microorganisms that can be handled
safely

22. Which of the following staining techniques is commonly used to detect Mycobacterium tuberculosis in
clinical samples?
a) Gram staining
b) Acid-fast staining

Page 36 of 92
c) Flagella staining
d) Capsule staining Answer: b) Acid-fast staining

23. What is the purpose of using MacConkey agar in microbiology laboratories?


a) To promote the growth of Gram-positive bacteria
b) To identify fungal species
c) To differentiate lactose-fermenting from non-lactose-fermenting bacteria
d) To test antibiotic susceptibility Answer: c) To differentiate lactose-fermenting from non-lactose-
fermenting bacteria

24. Which diagnostic method is most suitable for identifying the presence of viral RNA in a patient’s blood
sample?
a) Gram staining
b) Serological testing
c) PCR (Polymerase Chain Reaction)
d) Metagenomics Answer: c) PCR (Polymerase Chain Reaction)

25. What is the primary purpose of a Kirby-Bauer disk diffusion test?


a) Identifying bacterial species
b) Quantifying bacterial growth
c) Assessing antibiotic susceptibility
d) Detecting fungal infections Answer: c) Assessing antibiotic susceptibility

26. What is the primary advantage of using MALDI-TOF mass spectrometry for microbial identification?
a) It provides information about bacterial motility.
b) It allows for the visualization of bacterial colonies.
c) It provides rapid and accurate identification based on protein profiles.
d) It measures antibiotic sensitivity. Answer: c) It provides rapid and accurate identification based on
protein profiles.

27. Which of the following microscopes is most suitable for studying the internal structures of microorganisms
in detail?
a) Scanning electron microscope (SEM)
b) Transmission electron microscope (TEM)
c) Phase-contrast microscope
d) Confocal microscope Answer: b) Transmission electron microscope (TEM)

28. What does the term “aseptic technique” emphasize in microbiology laboratory practices?
a) Maintaining a sterile environment
b) Rapidly culturing microorganisms
c) Using selective media
d) Identifying microorganisms based on DNA sequencing Answer: a) Maintaining a sterile environment

29. Which of the following is NOT a common component of personal protective equipment (PPE) used in
microbiology laboratories?
a) Lab coat
b) Safety goggles
c) Hairnet
d) Office attire Answer: d) Office attire

Page 37 of 92
30. In microbiology, what is the primary function of using selective media?
a) To encourage the growth of all microorganisms
b) To promote rapid bacterial replication
c) To inhibit the growth of specific microorganisms
d) To enhance bacterial motility Answer: c) To inhibit the growth of specific microorganisms

31. What does the biosafety level (BSL) designation of a laboratory indicate?
a) The laboratory’s location
b) The level of funding provided to the laboratory
c) The types of microorganisms that can be safely handled in the laboratory
d) The laboratory’s cleanliness standards Answer: c) The types of microorganisms that can be safely
handled in the laboratory

Host Defenses
1. What is the primary function of the skin in host defense against microbial pathogens?
a) Temperature regulation
b) Sensation
c) Physical barrier
d) Nutrient absorption
Answer: c) Physical barrier

2. Which of the following is an example of an innate immune response mechanism?


a) Antibody production
b) T-cell activation
c) Phagocytosis
d) Memory cell formation
Answer: c) Phagocytosis

3. Which type of white blood cell is primarily responsible for the ingestion and destruction of
invading microbes?
a) T cells
b) B cells
c) Neutrophils
d) Platelets
Answer: c) Neutrophils

4. The complement system is a part of the:


a) Innate immune system
b) Adaptive immune system
c) Both innate and adaptive immune systems
d) None of the above
Answer: a) Innate immune system

5. Which of the following is an example of a specific (adaptive) immune response?


a) Inflammation
b) Fever
c) Antibody production

Page 38 of 92
d) Skin barrier
Answer: c) Antibody production

6. Immunization with a vaccine primarily activates which branch of the immune system?
a) Innate immune system
b) Adaptive immune system
c) Both innate and adaptive immune systems
d) None of the above
Answer: b) Adaptive immune system

7. What is the primary function of antibodies in the immune system?


a) Directly destroy pathogens
b) Promote phagocytosis
c) Inhibit complement activation
d) Regulate cytokine production
Answer: b) Promote phagocytosis

8. Which of the following is an example of a cellular component of the immune system?


a) Antibodies
b) Interferons
c) Natural killer cells
d) Complement proteins
Answer: c) Natural killer cells

9. Mucous membranes in the respiratory and digestive tracts are part of the host defense
mechanism primarily due to:
a) Antibody secretion
b) Coughing and sneezing reflexes
c) Mucus production and ciliary action
d) Enzyme secretion
Answer: c) Mucus production and ciliary action

10. Which of the following is a mechanism by which pathogens can evade host defenses?
a) Activation of phagocytes
b) Production of antibodies
c) Antigen presentation
d) Antigenic variation
Answer: d) Antigenic variation

11. What is the primary function of the spleen in the immune system?
a) Blood clotting
b) Red blood cell production
c) Filtering and monitoring blood for pathogens
d) Digestion of pathogens
Answer: c) Filtering and monitoring blood for pathogens

12. Which type of immune response is typically faster upon re-exposure to a previously
encountered pathogen?
a) Primary immune response

Page 39 of 92
b) Secondary immune response
c) Tertiary immune response
d) Innate immune response
Answer: b) Secondary immune response

13. Which of the following is an example of a chemical barrier in host defense?


a) Skin
b) Tears
c) Lymph nodes
d) Hair
Answer: b) Tears

14. The process by which immune cells recognize self from non-self is called:
a) Antigen presentation
b) Immune memory
c) Self-tolerance
d) Immunization
Answer: c) Self-tolerance

15. Which class of antibodies is most abundant in the bloodstream and is the first to be
produced during an immune response?
a) IgA
b) IgE
c) IgG
d) IgM
Answer: d) IgM

16. What is the primary function of cytotoxic T cells in the immune system?
a) Producing antibodies
b) Phagocytosis of pathogens
c) Directly killing infected cells
d) Regulating inflammation
Answer: c) Directly killing infected cells

17. Which of the following is an example of passive immunity?


a) Receiving a vaccination
b) Producing antibodies after an infection
c) Transferring antibodies from mother to child through breast milk
d) Activation of T cells
Answer: c) Transferring antibodies from mother to child through breast milk

18. What is the role of the thymus in the immune system?


a) Producing antibodies
b) Filtering blood for pathogens
c) Maturation of T cells
d) Producing mucus
Answer: c) Maturation of T cells

Page 40 of 92
19. Which of the following is a major function of the lymphatic system in host defense?
a) Oxygen transport
b) Nutrient absorption
c) Drainage of interstitial fluid and transport of immune cells
d) Regulation of body temperature
Answer: c) Drainage of interstitial fluid and transport of immune cells

20. Immunodeficiency disorders result in:


a) Enhanced immune responses
b) Weakened or compromised immune responses
c) Overproduction of antibodies
d) Autoimmune diseases
Answer: b) Weakened or compromised immune responses

21. Which of the following is NOT a component of the first line of defense in the immune
system?
a) Skin
b) Mucous membranes
c) Fever
d) Saliva
Answer: c) Fever

22. Which type of white blood cell is primarily responsible for presenting antigens to T cells
during an immune response?
a) Neutrophils
b) B cells
c) Macrophages
d) Natural killer cells
Answer: c) Macrophages

23. The process by which antibodies and complement proteins coat the surface of
pathogens, making them easier to be recognized and phagocytosed, is called:
a) Opsonization
b) Antigenic variation
c) Immunization
d) Autoimmunity
Answer: a) Opsonization

24. Which of the following is an example of a physical barrier in host defense?


a) Interferons
b) Cytokines
c) Cilia in the respiratory tract
d) Antibodies
Answer: c) Cilia in the respiratory tract

25. Which class of antibodies is involved in allergic reactions and defense against
parasites?
a) IgA
b) IgG

Page 41 of 92
c) IgE
d) IgM
Answer: c) IgE

26. Which of the following is a function of the lymph nodes in the immune system?
a) Filtering and monitoring blood
b) Maturation of T cells
c) Production of red blood cells
d) Digestion of pathogens
Answer: b) Maturation of T cells

27. Which type of immunity is acquired through vaccination or exposure to a disease-


causing pathogen?
a) Innate immunity
b) Passive immunity
c) Natural immunity
d) Active immunity
Answer: d) Active immunity

28. What is the primary role of helper T cells in the immune system?
a) Directly kill infected cells
b) Produce antibodies
c) Coordinate immune responses
d) Phagocytose pathogens
Answer: c) Coordinate immune responses

29. Which of the following is an example of a chemical mediator of inflammation?


a) Antibodies
b) Histamine
c) Neutrophils
d) T cells
Answer: b) Histamine

30. Autoimmune diseases occur when the immune system:


a) Fails to respond to pathogens
b) Attacks the body’s own tissues
c) Is overactive in response to infections
d) Produces too many antibodies
Answer: b) Attacks the body’s own tissues

Blood collection tube colors and tests


1. Which color of bottle is commonly used for blood collection in clinical laboratories?
a) Red
b) Green
c) Blue
d) Yellow
Answer: a) Red

Page 42 of 92
2. What anticoagulant is typically found in red-topped blood collection tubes?
a) Heparin
b) EDTA
c) Sodium citrate
d) None
Answer: d) None

3. Which color of bottle is often used for coagulation testing in clinical laboratories?
a) Green
b) Purple
c) Blue
d) Gray
Answer: c) Blue

4. What anticoagulant is commonly found in blue-topped blood collection tubes?


a) Heparin
b) EDTA
c) Sodium citrate
d) ACD
Answer: c) Sodium citrate

5. Which color of bottle is used for glucose testing in clinical laboratories?


a) Yellow
b) Red
c) Gray
d) Lavender
Answer: a) Yellow

6. What anticoagulant is typically found in yellow-topped blood collection tubes?


a) Heparin
b) EDTA
c) Sodium citrate
d) ACD
Answer: a) Heparin

7. Which color of bottle is used for blood culture specimens?


a) Pink
b) Blue
c) Lavender
d) Gray
Answer: a) Pink

8. What anticoagulant is commonly found in pink-topped blood collection tubes for blood
cultures?
a) Heparin
b) EDTA
c) Sodium citrate
d) SPS (Sodium Polyanethol Sulfonate)
Answer: d) SPS (Sodium Polyanethol Sulfonate)

Page 43 of 92
9. Which color of bottle is used for hematology testing, including complete blood counts
(CBC)?
a) Green
b) Red
c) Lavender
d) Gray
Answer: c) Lavender

10. What anticoagulant is typically found in lavender-topped blood collection tubes?


a) Heparin
b) EDTA
c) Sodium citrate
d) ACD
Answer: b) EDTA

11. Which color of bottle is commonly used for chemistry tests, such as electrolytes and
liver function tests, in clinical laboratories?
a) Green
b) Red
c) Lavender
d) Gray
Answer: b) Red

12. What anticoagulant is typically found in green-topped blood collection tubes?


a) Heparin
b) EDTA
c) Sodium citrate
d) Lithium heparin
Answer: d) Lithium heparin

13. Which color of bottle is used for blood gas analysis, including pH and pCO2
measurements?
a) Green
b) Blue
c) Brown
d) Gray
Answer: c) Brown

14. What anticoagulant is commonly found in brown-topped blood collection tubes for blood
gas analysis?
a) Heparin
b) EDTA
c) Sodium citrate
d) Heparin with gel separator
Answer: d) Heparin with gel separator

15. Which color of bottle is used for trace element testing, such as zinc and copper levels?
a) Yellow
b) Gray

Page 44 of 92
c) Black
d) Royal blue
Answer: d) Royal blue

16. What anticoagulant or additive is typically found in royal blue-topped blood collection
tubes for trace element testing?
a) Heparin
b) EDTA
c) Sodium citrate
d) None (no anticoagulant)
Answer: d) None (no anticoagulant)

17. Which color of bottle is used for blood typing and compatibility testing?
a) Blue
b) Pink
c) Gray
d) Orange
Answer: b) Pink

18. What anticoagulant is commonly found in pink-topped blood collection tubes for blood
typing and compatibility testing?
a) Heparin
b) EDTA
c) Sodium citrate
d) ACD
Answer: c) Sodium citrate

19. Which color of bottle is used for collecting samples for drug testing, including toxicology
screenings?
a) Green
b) Red
c) Gray
d) Gold (Tiger top)
Answer: d) Gold (Tiger top)

20. What additive is typically found in gold (Tiger top) blood collection tubes for drug
testing?
a) Heparin
b) EDTA
c) Gel separator with clot activator
d) Sodium citrate
Answer: c) Gel separator with clot activator

21. Which color of bottle is commonly used for collecting samples for serology tests, such
as HIV or syphilis tests?
a) Red
b) Blue
c) Yellow

Page 45 of 92
d) Black
Answer: c) Yellow

22. What anticoagulant is typically found in yellow-topped blood collection tubes for
serology testing?
a) Heparin
b) EDTA
c) Sodium citrate
d) ACD
Answer: d) ACD (Acid Citrate Dextrose)

23. Which color of bottle is used for erythrocyte sedimentation rate (ESR) testing?
a) Green
b) Red
c) Purple
d) Black
Answer: c) Purple

24. What anticoagulant is commonly found in purple-topped blood collection tubes for ESR
testing?
a) Heparin
b) EDTA
c) Sodium citrate
d) ACD
Answer: b) EDTA

25. Which color of bottle is used for collecting samples for blood bank crossmatching?
a) Green
b) Red
c) Pink
d) Black
Answer: b) Red

26. What type of additive is typically found in blood bank collection tubes for
crossmatching?
a) Heparin
b) EDTA
c) Sodium citrate
d) No additive, just a clot activator
Answer: d) No additive, just a clot activator

27. Which color of bottle is used for special coagulation tests, such as D-dimer or
fibrinogen assays?
a) Green
b) Blue
c) Light blue
d) Gray
Answer: c) Light blue

Page 46 of 92
28. What anticoagulant is commonly found in light blue-topped blood collection tubes for
special coagulation tests?
a) Heparin
b) EDTA
c) Sodium citrate
d) Sodium polyanethol sulfonate (SPS)
Answer: c) Sodium citrate

29. Which color of bottle is used for collecting samples for lead testing?
a) Green
b) Red
c) Lavender
d) Royal blue
Answer: d) Royal blue

30. What additive is typically found in royal blue-topped blood collection tubes for lead
testing?
a) Heparin
b) EDTA
c) Sodium citrate
d) None (no additive)
Answer: d) None (no additive)

31. Which color of bottle is typically used for collecting samples for toxicology testing, such
as drug screens?
a) Green
b) Red
c) Gray
d) Orange
Answer: d) Orange

32. What additive is commonly found in orange-topped blood collection tubes for toxicology
testing?
a) Heparin
b) EDTA
c) Gel separator with clot activator
d) Sodium citrate
Answer: c) Gel separator with clot activator

33. Which color of bottle is used for collecting samples for blood ethanol (alcohol) testing?
a) Green
b) Red
c) Gray
d) Light blue
Answer: d) Light blue

34. What anticoagulant or additive is typically found in light blue-topped blood collection
tubes for blood ethanol testing?
a) Heparin

Page 47 of 92
b) EDTA
c) Sodium citrate
d) Sodium fluoride
Answer: d) Sodium fluoride

35. Which color of bottle is used for collecting samples for arterial blood gas (ABG)
analysis?
a) Green
b) Blue
c) Brown
d) Gray
Answer: b) Blue

36. What anticoagulant or preservative is commonly found in blood collection tubes for
arterial blood gas (ABG) analysis?
a) Heparin
b) EDTA
c) Sodium citrate
d) Heparin with gel separator
Answer: a) Heparin

37. Which color of bottle is used for collecting samples for immunology tests, such as
autoimmune or allergy panels?
a) Red
b) Blue
c) Yellow
d) Gold (Tiger top)
Answer: b) Blue

38. What anticoagulant is typically found in blue-topped blood collection tubes for
immunology testing?
a) Heparin
b) EDTA
c) Sodium citrate
d) None (no anticoagulant)
Answer: d) None (no anticoagulant)

39. Which color of bottle is used for collecting samples for cytogenetics or molecular
genetics testing?
a) Red
b) Blue
c) Yellow
d) Gray
Answer: d) Gray

40. What additive is commonly found in gray-topped blood collection tubes for cytogenetics
or molecular genetics testing?
a) Heparin
b) EDTA

Page 48 of 92
c) Sodium citrate
d) Special preservative solutions
Answer: d) Special preservative solutions

Pathogenesis
Question 1: What is the term for the ability of a microorganism to cause disease?

A) Virulence
B) Immunity
C) Sterility
D) Pathogenicity

Answer: A) Virulence

Question 2: Which of the following is NOT a component of the Koch’s postulates?

A) The microorganism must be isolated from a diseased individual and grown in pure culture.
B) The microorganism must cause disease when introduced into a healthy individual.
C) The microorganism must be visible under a light microscope.
D) The microorganism must be re-isolated from the experimentally infected individual.

Answer: C) The microorganism must be visible under a light microscope.

Question 3: Which term refers to the ability of a microorganism to cause severe disease in a
host?

A) Infectivity
B) Virulence
C) Pathogenicity
D) Contagiousness

Answer: B) Virulence

Question 4: Which of the following is an example of an exotoxin?

A) Lipopolysaccharide (LPS)
B) Streptolysin S
C) Lipoteichoic acid
D) Peptidoglycan

Answer: B) Streptolysin S

Question 5: How do endotoxins differ from exotoxins?

A) Endotoxins are proteins released by bacteria, while exotoxins are lipopolysaccharides.


B) Endotoxins are produced by Gram-positive bacteria, while exotoxins are produced by
Gram-negative bacteria.
C) Endotoxins are heat-labile, while exotoxins are heat-stable.

Page 49 of 92
D) Endotoxins are structural components of the outer membrane of Gram-negative bacteria,
while exotoxins are secreted proteins.

Answer: D) Endotoxins are structural components of the outer membrane of Gram-negative


bacteria, while exotoxins are secreted proteins.

Question 6: Which type of pathogen can establish an infection only if the host’s immune
system is compromised?

A) Obligate intracellular pathogen


B) Facultative intracellular pathogen
C) Opportunistic pathogen
D) Endotoxin-producing pathogen

Answer: C) Opportunistic pathogen

Question 7: Which of the following is an example of a direct transmission route for infectious
diseases?

A) Ingesting contaminated food


B) Breathing in respiratory droplets from a cough or sneeze
C) Getting bitten by an infected mosquito
D) Touching a surface that has infectious particles and then touching the face

Answer: B) Breathing in respiratory droplets from a cough or sneeze

Question 8: What is the primary function of pili (fimbriae) in bacterial pathogenesis?

A) To facilitate bacterial movement


B) To aid in bacterial replication
C) To promote adhesion to host cells
D) To protect bacteria from antibiotics

Answer: C) To promote adhesion to host cells

Question 9: Which term describes a state of unresponsiveness to antigens from one’s own
body?

A) Immunity
B) Allergy
C) Autoimmunity
D) Tolerance

Answer: D) Tolerance

Question 10: Which of the following is a mechanism that bacteria use to evade the host
immune system?

A) Upregulating toll-like receptors


B) Producing antibodies against host cells

Page 50 of 92
C) Antigenic variation
D) Enhancing phagocytosis

Answer: C) Antigenic variation

Question 11: Which of the following is an example of a latent infection?

A) Influenza
B) Tuberculosis
C) Common cold
D) Streptococcal throat infection

Answer: B) Tuberculosis

Question 12: How does a capsule contribute to bacterial pathogenicity?

A) It enhances bacterial movement


B) It facilitates DNA exchange between bacteria
C) It helps bacteria evade phagocytosis
D) It acts as a source of nutrients for bacteria

Answer: C) It helps bacteria evade phagocytosis

Question 13: What is a zoonotic disease?

A) A disease caused by viruses only


B) A disease that primarily affects plants
C) A disease transmitted from animals to humans
D) A disease that spreads through waterborne transmission

Answer: C) A disease transmitted from animals to humans

Question 14: Which of the following is a vector-borne transmission route for infectious
diseases?

A) Inhaling respiratory droplets


B) Direct contact with the infected person’s skin
C) Consuming contaminated food
D) Being bitten by infected ticks

Answer: D) Being bitten by infected ticks

Question 15: What is the primary function of MHC molecules in the immune response?

A) To directly attack pathogens


B) To recognize pathogen-associated molecular patterns (PAMPs)
C) To present antigens to immune cells
D) To neutralize toxins produced by pathogens

Answer: C) To present antigens to immune cells

Page 51 of 92
Question 16: Which of the following is a virulence factor that allows bacteria to resist
phagocytosis by immune cells?

A) Antibodies
B) Complement proteins
C) Phagolysosomes
D) Capsule formation

Answer: D) Capsule formation

Question 17: What is the role of superantigens in bacterial infections?

A) They inhibit bacterial replication


B) They trigger an exaggerated immune response
C) They directly kill host cells
D) They prevent the release of endotoxins

Answer: B) They trigger an exaggerated immune response

Question 18: Which type of toxin causes the symptoms of fever, inflammation, and shock in
response to Gram-negative bacterial infections?

A) Exotoxins
B) Enterotoxins
C) Lipopolysaccharides (LPS)
D) Endotoxins

Answer: D) Endotoxins

Question 19: Which of the following is an example of a chronic infection?

A) Measles
B) Chickenpox
C) Acute bacterial meningitis
D) HIV/AIDS

Answer: D) HIV/AIDS

Question 20: How do vaccines primarily protect individuals from infectious diseases?

A) By directly killing the pathogens


B) By stimulating a specific immune response against the pathogens
C) By blocking all types of pathogen entry
D) By neutralizing toxins produced by pathogens

Answer: B) By stimulating a specific immune response against the pathogens

Question 21: Which of the following best describes a reservoir in the context of infectious
diseases?

Page 52 of 92
A) An organism that carries a pathogen without showing symptoms of the disease
B) A place where pathogens are produced in large quantities
C) A laboratory where microbial cultures are maintained
D) A region with a high prevalence of a specific disease

Answer: A) An organism that carries a pathogen without showing symptoms of the disease

Question 22: Which of the following is an example of a protozoan pathogen?

A) Escherichia coli (E. coli)


B) Staphylococcus aureus
C) Plasmodium falciparum
D) Streptococcus pyogenes

Answer: C) Plasmodium falciparum

Question 23: What is the term for a microorganism’s genetic makeup, which can influence its
ability to cause disease?

A) Virulence factors
B) Phenotype
C) Pathogenicity islands
D) Genotype

Answer: D) Genotype

Question 24: Which of the following is an example of a fomite?

A) A mosquito carrying a virus


B) A contaminated piece of fruit
C) An infected person sneezing
D) Airborne bacteria

Answer: B) A contaminated piece of fruit

Question 25: What is the primary role of the mucociliary escalator in preventing respiratory
infections?

A) Producing antibodies against pathogens


B) Trapping and removing inhaled pathogens and debris
C) Destroying pathogens using enzymes
D) Stimulating fever as an immune response

Answer: B) Trapping and removing inhaled pathogens and debris

Question 26: Which type of toxin targets nerve cells and can lead to symptoms like muscle
paralysis?

A) Enterotoxin
B) Cytotoxin

Page 53 of 92
C) Neurotoxin
D) Hemolysin

Answer: C) Neurotoxin

Question 27: What is the mechanism of action of antiseptics?

A) Killing bacteria inside the body


B) Enhancing the immune response
C) Killing bacteria on living tissues
D) Neutralizing bacterial toxins

Answer: C) Killing bacteria on living tissues

Question 28: What role do cytokines play in the immune response to infection?

A) They are structural components of antibodies


B) They help neutralize bacterial toxins
C) They regulate immune cell communication and response
D) They are enzymes that break down bacterial cell walls

Answer: C) They regulate immune cell communication and response

Question 29: Which of the following is an example of a systemic infection?

A) Athlete’s foot
B) Urinary tract infection
C) Tuberculosis
D) Strep throat

Answer: C) Tuberculosis

Question 30: Which of the following is a common portal of entry for pathogens?

A) Healthy skin
B) Intact mucous membranes
C) Respiratory tract
D) Intact digestive system

Answer: C) Respiratory tract

Question 31: Which type of immune response involves antibodies produced by B cells?

A) Cell-mediated immunity
B) Innate immunity
C) Humoral immunity
D) Passive immunity

Answer: C) Humoral immunity

Question 32: Which of the following is an example of a viral pathogen?

Page 54 of 92
A) Candida albicans
B) Escherichia coli
C) Influenza virus
D) Mycobacterium tuberculosis

Answer: C) Influenza virus

Question 33: What is the primary function of the complement system in the immune
response?

A) Producing antibodies
B) Destroying infected host cells
C) Enhancing phagocytosis
D) Promoting inflammation

Answer: B) Destroying infected host cells

Question 34: What is the role of adhesins in bacterial pathogenesis?

A) To neutralize toxins
B) To prevent phagocytosis
C) To stimulate antibody production
D) To promote attachment to host cells

Answer: D) To promote attachment to host cells

Question 35: Which of the following is a characteristic of acute infections?

A) Long-lasting and persistent symptoms


B) Slow onset and gradual progression
C) Gradual decline in symptoms over time
D) Rapid onset and short duration

Answer: D) Rapid onset and short duration

Question 36: What is the term for a population in which a specific disease regularly spreads?

A) Pandemic
B) Endemic
C) Epidemic
D) Outbreak

Answer: B) Endemic

Question 37: Which of the following is an example of a Gram-negative bacterial pathogen?

A) Staphylococcus aureus
B) Streptococcus pyogenes
C) Escherichia coli
D) Clostridium botulinum

Page 55 of 92
Answer: C) Escherichia coli

Question 38: What is the role of neutrophils in the immune response?

A) Producing antibodies
B) Regulating T-cell responses
C) Phagocytosing pathogens and debris
D) Recognizing antigens on infected cells

Answer: C) Phagocytosing pathogens and debris

Question 39: Which of the following is an example of a foodborne transmission route for
infectious diseases?

A) Inhalation of airborne pathogens


B) Contact with contaminated surfaces
C) Transmission through sexual contact
D) Consumption of contaminated water or food

Answer: D) Consumption of contaminated water or food

Question 40: What is the primary function of memory cells in the immune response?

A) To produce antibodies
B) To initiate inflammation
C) To recognize and respond to repeat infections
D) To differentiate into various immune cell types

Answer: C) To recognize and respond to repeat infections

Question 41: Endotoxins are chemically?

A) Lipopolysaccharide
B) Polypeptide

Question 42: Exotoxins are chemically?

A) lipopolysaccharide
B) Polypeptide

Question 43: Traveler’s diarrhea (Watery, none bloody diarrhea) caused by the?

A) E.coli
B) Labile toxin
C) Both

Question 44: Bloody diarrhea associated with undercooked hamburgers caused by which
kinds of toxin?

Page 56 of 92
A) Verotoxin
B) Shiga Like Toxin
C) BOTH

Question 45: What is the most important cause of SEPTIC SHOCK?

A) Endotoxin
B) Exotoxin

Question 46: Which toxin produced by both gram positive and gram negative?

A) Endotoxin
B) Exotoxin

Question 47: Which toxin is produced by ONLY gram-negative bacteria?

A) Endotoxin
B) Exotoxin

Question 48: Hand washing is an important means of interrupting the chain of transmission
from ONE person to ANOTHER?

A) Saph Aureus
B) Strep Agalactiae

Classification of bacteria
Question 1: Which of the following bacterial groups possess a thick layer of peptidoglycan in
their cell walls, making them Gram-positive?
A) Proteobacteria
B) Cyanobacteria
C) Firmicutes
D) Spirochetes

Answer: C) Firmicutes

Question 2: Bacteria with a thin layer of peptidoglycan in their cell walls, which stains pink in
Gram staining, are referred to as:
A) Gram-negative
B) Gram-positive
C) Gram-neutral
D) Gram-variable

Answer: A) Gram-negative

Question 3: The bacterial group that includes species with diverse shapes such as cocci,
bacilli, and spiral forms, and possess an outer membrane in their cell wall structure, is:
A) Actinobacteria
B) Firmicutes

Page 57 of 92
C) Proteobacteria
D) Cyanobacteria

Answer: C) Proteobacteria

Question 4: Which bacterial phylum comprises organisms that are often found in extreme
environments, have a unique cell wall structure, and include the genus Thermus?
A) Chloroflexi
B) Thermotogae
C) Aquificae
D) Deinococcus-Thermus

Answer: D) Deinococcus-Thermus

Question 5: The bacterial group known for their branching filaments, often forming complex
mycelium-like structures, and having a high G+C content in their DNA, is:
A) Cyanobacteria
B) Spirochetes
C) Actinobacteria
D) Chlamydiae

Answer: C) Actinobacteria

Question 6: Which of the following bacterial groups is characterized by a unique cell wall
structure containing pseudomurein, and they are often found in extreme environments like hot
springs and hydrothermal vents?
A) Firmicutes
B) Spirochetes
C) Crenarchaeota
D) Thaumarchaeota

Answer: C) Crenarchaeota

Question 7: The bacterial group responsible for nitrogen fixation and often found in root
nodules of leguminous plants belongs to which phylum?
A) Cyanobacteria
B) Proteobacteria
C) Spirochetes
D) Chlamydiae

Answer: B) Proteobacteria

Question 8: The genus Borrelia which causes diseases like Lyme disease is a member of
which bacterial group?
A) Firmicutes
B) Actinobacteria
C) Spirochetes
D) Thermotogae

Answer: C) Spirochetes

Page 58 of 92
Question 9: Which bacterial phylum includes species that lack a peptidoglycan cell wall and
are characterized by a unique cell membrane composed of ether-linked lipids?
A) Planctomycetes
B) Fusobacteria
C) Bacteroidetes
D) Chlamydiae

Answer: A) Planctomycetes

Question 10: The bacterial group responsible for many sexually transmitted infections and
includes the genus Chlamydia is:
A) Firmicutes
B) Actinobacteria
C) Spirochetes
D) Chlamydiae

Answer: D) Chlamydiae

Question 11: Bacteria belonging to the phylum Cyanobacteria are known for:
A) Having no cell walls
B) Being Gram-negative
C) Performing photosynthesis
D) Being thermophilic

Answer: C) Performing photosynthesis

Question 12: The bacterial phylum Spirochaetes is characterized by:


A) Formation of endospores
B) Helical or spiral-shaped cells
C) Being acidophilic
D) Lack of cell walls

Answer: B) Helical or spiral-shaped cells

Question 13: Which bacterial group is responsible for causing tuberculosis and leprosy?
A) Actinobacteria
B) Cyanobacteria
C) Chlamydiae
D) Thermotogae

Answer: A) Actinobacteria

Question 14: The bacterial phylum Chloroflexi includes species that are commonly found in:
A) Extreme thermophilic environments
B) Oxygen-rich environments
C) Anaerobic environments
D) Saline environments

Answer: A) Extreme thermophilic environments

Page 59 of 92
Question 15: Which bacterial phylum includes species responsible for producing methane as
a metabolic byproduct?
A) Bacteroidetes
B) Thermotogae
C) Firmicutes
D) Euryarchaeota

Answer: D) Euryarchaeota

Question 16: Bacteria of the phylum Fusobacteria are often associated with infections
involving:
A) Gastrointestinal tract
B) Respiratory tract
C) Urinary tract
D) Oral cavity

Answer: D) Oral cavity

Question 17: Which bacterial group includes the genus Mycoplasma known for their lack of a
cell wall?
A) Firmicutes
B) Actinobacteria
C) Spirochetes
D) Tenericutes

Answer: D) Tenericutes

Question 18: The bacterial group Planctomycetes is distinct for possessing:


A) Intracellular membrane compartments
B) Extremely thick cell walls
C) Multiple flagella
D) Photosynthetic pigments

Answer: A) Intracellular membrane compartments

Question 19: The phylum Aquificae includes thermophilic bacteria that generate energy
through:
A) Photosynthesis
B) Methanogenesis
C) Sulfur oxidation
D) Hydrogen oxidation

Answer: D) Hydrogen oxidation

Question 20: The bacterial phylum Bacteroidetes is commonly found in which type of
environments?
A) Acidic hot springs
B) Extreme halophilic environments
C) Marine and freshwater habitats
D) Deep-sea hydrothermal vents
Page 60 of 92
Answer: C) Marine and freshwater habitats

Question 21: Gram-positive, cocci, cause abscesses of skin are the characteristics of which
bacteria.
A) Staphylococcus
B) Streptococcus
C) E.coli

Question 22: Gram-positive, aerobic, spore-forming rods are the characteristics of which
bacteria.
A) Bacilius
B) Clostridium
C) E.coli

Question 23: Gram-negative, cocci, cause gonorrhea are the characteristics of which
bacteria?
A) Staphylococcus
B) Neisseria
C) E.coli

Question 24: Gram-negative, rod, respiratory organisms cause whooping cough are the
characteristics of which bacteria?
A) Hemophilus
B) Bordetella
C) E.coli

Question 25: Gram-negative, rod, enteric related cause typhoid fever are the characteristics
of which bacteria?
A) Shagella
B) Salmonella
C) Proteus

Question 26: Gram-negative, curved shape, cause peptic ulcer, gastric ulcer are the
characteristics of which bacteria?
A) Campylobactor
B) Helicobacter
C) E.coli

Question 27: Obligatory intracellular parasites, cause Q-fever, and typhus are the
characteristics of which bacteria?
A) Rickettsia
B) Chlamydia
C) None

Question 28: Wallless cells, cause Pneumonia are the characteristics of which bacteria?
A) Mycoplasma
B) Treponema
C) Leptospira

Page 61 of 92
mutation, conjugation, transduction,
transformation
Question 1: What type of mutation involves the insertion or deletion of a single
nucleotide, leading to a shift in the reading frame of the gene?

A) Silent mutation
B) Missense mutation
C) Nonsense mutation
D) Frameshift mutation

Answer: D) Frameshift mutation

Question 2: Which of the following is NOT a cause of mutations?

A) Exposure to mutagenic chemicals


B) Spontaneous errors during DNA replication
C) Repair mechanisms
D) Radiation

Answer: C) Repair mechanisms

Question 3: Which of the following mutations results in a change of a single nucleotide


that leads to the substitution of one amino acid in the protein?

A) Silent mutation
B) Nonsense mutation
C) Missense mutation
D) Frameshift mutation

Answer: C) Missense mutation

Question 4: Mutations that occur in somatic cells are responsible for:

A) Genetic diseases passed to offspring


B) The formation of gametes
C) Cancer and certain genetic diseases in the individual
D) None of the above

Answer: C) Cancer and certain genetic diseases in the individual

Question 5: Which type of mutagen is known to cause physical alterations to DNA


structure?

A) Chemical mutagen
B) Radiation mutagen
C) Spontaneous mutagen
D) Replicative mutagen

Page 62 of 92
Answer: B) Radiation mutagen

Question 6: A mutation that changes a codon encoding an amino acid to a stop codon
is called:

A) Silent mutation
B) Missense mutation
C) Nonsense mutation
D) Frameshift mutation

Answer: C) Nonsense mutation

Question 7: Which repair mechanism specifically targets incorrect base pairs that
occur due to errors during DNA replication?

A) Mismatch repair
B) Nucleotide excision repair
C) Base excision repair
D) Direct repair

Answer: A) Mismatch repair

Question 8: Which of the following is an example of a point mutation?

A) Deletion of a large segment of DNA


B) Duplication of a gene
C) Substitution of one nucleotide for another
D) Exchange of genetic material between two chromosomes

Answer: C) Substitution of one nucleotide for another

Question 9: The lac operon is an example of which type of gene regulation?

A) Inducible operon
B) Repressible operon
C) Constitutive operon
D) Nonsense operon

Answer: A) Inducible operon

Question 10: What is the term for the process by which bacteria acquire new genetic
material from their environment or from other bacteria?

A) Transduction
B) Transformation
C) Conjugation
D) Replication

Answer: B) Transformation

Page 63 of 92
Question 11: Which of the following is a heritable mutation that occurs in gametes and
can be passed on to offspring?

A) Somatic mutation
B) Spontaneous mutation
C) Germline mutation
D) Induced mutation

Answer: C) Germline mutation

Question 12: A mutation that has no detectable effect on the phenotype of an organism
is called:

A) Silent mutation
B) Nonsense mutation
C) Missense mutation
D) Frameshift mutation

Answer: A) Silent mutation

Question 13: The Ames test is used to:

A) Detect carcinogens by their mutagenic effects on bacteria


B) Determine the rate of spontaneous mutations
C) Assess the repair mechanisms in DNA
D) Study the impact of frameshift mutations

Answer: A) Detect carcinogens by their mutagenic effects on bacteria

Question 14: Which of the following is a type of mutagenic chemical that mimics a DNA
base and can be incorporated into the DNA strand during replication, causing mistakes
in base pairing?

A) UV radiation
B) X-rays
C) Intercalating agent
D) Alkylating agent

Answer: C) Intercalating agent

Question 15: A mutation in a gene that leads to the loss of its function is termed:

A) Gain-of-function mutation
B) Nonsense mutation
C) Null mutation
D) Spontaneous mutation

Answer: C) Null mutation

Question 16: The process of DNA repair that involves removing a segment of damaged
DNA and replacing it with the correct sequence is known as:
Page 64 of 92
A) Mismatch repair
B) Base excision repair
C) Nucleotide excision repair
D) Direct repair

Answer: C) Nucleotide excision repair

Question 17: What type of mutation occurs when one or more nucleotides are added or
deleted from the DNA sequence, causing a shift in the reading frame?

A) Silent mutation
B) Missense mutation
C) Nonsense mutation
D) Frameshift mutation

Answer: D) Frameshift mutation

Question 18: The trp operon is an example of which type of gene regulation?

A) Inducible operon
B) Repressible operon
C) Constitutive operon
D) Nonsense operon

Answer: B) Repressible operon

Question 19: Which repair mechanism involves the direct reversal of DNA damage,
such as the removal of alkyl groups from DNA bases?

A) Mismatch repair
B) Base excision repair
C) Nucleotide excision repair
D) Direct repair

Answer: D) Direct repair

Question 20: Mutations in the BRCA1 and BRCA2 genes are associated with an
increased risk of:

A) Heart disease
B) Diabetes
C) Breast and ovarian cancer
D) Lung cancer

Answer: C) Breast and ovarian cancer

Question 21: In bacterial conjugation, which structure is responsible for transferring


genetic material from the donor bacterium to the recipient bacterium?

A) Plasmid
B) Pilus
Page 65 of 92
C) Capsule
D) Flagellum

Answer: B) Pilus

Question 22: Which type of genetic material is commonly transferred through bacterial
conjugation?

A) Chromosomal DNA
B) Ribosomal RNA
C) Mitochondrial DNA
D) Plasmid DNA

Answer: D) Plasmid DNA

Question 23: In conjugation, which bacterium is typically referred to as the “donor”


bacterium?

A) The bacterium that receives genetic material


B) The bacterium that donates genetic material
C) A bacterium that remains neutral
D) A bacterium that initiates transformation

Answer: B) The bacterium that donates genetic material

Question 24: What is transduction in bacterial genetics?

A) The direct transfer of genetic material from one bacterium to another


B) The incorporation of foreign DNA into a bacterial genome
C) The transfer of genetic material between bacteria through phages
D) The exchange of genetic material through bacterial conjugation

Answer: C) The transfer of genetic material between bacteria through phages

Question 25: Which of the following is a bacteriophage-mediated transduction


process?

A) Transformation
B) Conjugation
C) Horizontal gene transfer
D) Viral vectors transferring DNA

Answer: D) Viral vectors transferring DNA

Question 26: Which of the following is a characteristic of specialized transduction?

A) It involves the transfer of any bacterial genes to a recipient bacterium.


B) It occurs during bacterial conjugation.
C) It results from the accidental packaging of host DNA into a phage particle.
D) It only transfers specific bacterial genes located near the phage integration site.

Page 66 of 92
Answer: D) It only transfers specific bacterial genes located near the phage integration
site.

Transformation:

Question 27: What is bacterial transformation?

A) The process of transferring genetic material through pili.


B) The exchange of genetic material between two bacterial cells.
C) The uptake and incorporation of foreign DNA into a bacterial cell.
D) The process of transferring genetic material through bacteriophages.

Answer: C) The uptake and incorporation of foreign DNA into a bacterial cell.

Question 28: Which molecule is crucial for bacterial transformation to occur?

A) Plasmid
B) Capsule
C) Pilus
D) Competence factor

Answer: D) Competence factor

Question 29: What is a competent bacterium in the context of bacterial transformation?

A) A bacterium that has gained resistance to antibiotics.


B) A bacterium that can donate genetic material via conjugation.
C) A bacterium that is able to take up and incorporate foreign DNA.
D) A bacterium that is resistant to phage infection.

Answer: C) A bacterium that is able to take up and incorporate foreign DNA.

Question 30: What is the primary function of the F (fertility) plasmid in bacterial
conjugation?

A) To enhance bacterial motility


B) To provide antibiotic resistance to the recipient bacterium
C) To serve as a template for DNA replication
D) To carry the genetic information needed for conjugation

Answer: D) To carry the genetic information needed for conjugation

Question 31: Which bacterial genus is known for its ability to undergo conjugation,
leading to the transfer of antibiotic-resistance genes?

A) Escherichia
B) Bacillus
C) Streptococcus
D) Clostridium

Answer: A) Escherichia

Page 67 of 92
Question 32: In generalized transduction, which type of genetic material can be
transferred from one bacterium to another?

A) Only plasmid DNA


B) Only chromosomal DNA
C) Any bacterial genetic material, regardless of origin
D) Only transposons

Answer: C) Any bacterial genetic material, regardless of origin

Question 33: Which term refers to the process where bacterial genes located near the
integrated phage DNA are transferred to a recipient bacterium via a transducing phage
particle?

A) Specialized transduction
B) Generalized transduction
C) Conjugative transduction
D) Fragmented transduction

Answer: A) Specialized transduction

Question 34: In natural transformation, what is required for the bacterial cell to uptake
foreign DNA?

A) Contact with a conjugative pilus


B) Physical attachment to a bacteriophage
C) Formation of a F pilus
D) Competence, a physiological state enabling DNA uptake

Answer: D) Competence, a physiological state enabling DNA uptake

Question 35: Which of the following bacterial species is well known for its ability to
undergo natural transformation?

A) Staphylococcus aureus
B) Mycobacterium tuberculosis
C) Streptococcus pneumoniae
D) Clostridium botulinum

Answer: C) Streptococcus pneumoniae

Question 36: What term describes the genetic material taken up from the environment
by a bacterial cell and incorporated into its genome?

A) Conjugation
B) Transduction
C) Transformation
D) Replication

Answer: C) Transformation

Page 68 of 92
Question 37: Which bacterial group is commonly associated with the ability to transfer
antibiotic resistance genes through conjugation?

A) Gram-positive bacteria
B) Acid-fast bacteria
C) Obligate anaerobic bacteria
D) Cyanobacteria

Answer: A) Gram-positive bacteria

Question 38: During bacterial conjugation, which of the following molecules is


responsible for the physical connection between donor and recipient bacteria?

A) Plasmid
B) Flagellum
C) Pilus
D) Capsule

Answer: C) Pilus

Question 39: Which type of transduction involves the accidental packaging of host
DNA into a phage particle during the lytic cycle?

A) Generalized transduction
B) Specialized transduction
C) Conjugative transduction
D) Directed transduction

Answer: A) Generalized transduction

Question 40: What is the term for a bacterial cell that is susceptible to being infected
by a transducing phage and receiving foreign DNA?

A) Lytic cell
B) Non-receptive cell
C) Competent cell
D) Conjugative cell

Answer: C) Competent cell

Question 41: Which discovery is often credited to Frederick Griffith’s experiment using
Streptococcus pneumoniae?

A) Bacterial conjugation
B) Bacterial transduction
C) Bacterial transformation
D) Bacterial replication

Answer: C) Bacterial transformation

Page 69 of 92
Question 42: In bacterial transformation, what is the process that involves the uptake
of naked DNA fragments from the environment?

A) Uptake and incorporation


B) Transduction
C) Conjugation
D) Uptake and integration

Answer: A) Uptake and incorporation

Question 43: What does the term “competence” refer to in the context of bacterial
transformation?

A) The ability of bacteria to produce toxins


B) The capacity of bacteria to replicate their DNA
C) The physiological state enabling bacteria to uptake and incorporate foreign DNA
D) The process of transferring genetic material through pili

Answer: C) The physiological state enabling bacteria to uptake and incorporate foreign
DNA

Question 44: What is the basic unit of heredity that carries genetic information?

a) Protein
b) Amino Acid
c) Nucleotide
d) Carbohydrate

Answer: c) Nucleotide

Question 45: Which scientist is known as the “Father of Genetics” for his work on pea
plants?

a) Isaac Newton
b) Gregor Mendel
c) Charles Darwin
d) Louis Pasteur

Answer: b) Gregor Mendel

Question 46: What is the term for the specific location of a gene on a chromosome?

a) Allele
b) Locus
c) Genotype
d) Phenotype

Answer: b) Locus

Question 47: A trait that is expressed only when an individual has two identical alleles for a
particular gene is called:
Page 70 of 92
a) Dominant trait
b) Recessive trait
c) Co-dominant trait
d) Incomplete dominance

Answer: b) Recessive trait

Question 48: Which genetic disorder is characterized by a missing or non-functional X


chromosome in females, leading to developmental and cognitive issues?

a) Down syndrome
b) Hemophilia
c) Turner syndrome
d) Cystic fibrosis

Answer: c) Turner syndrome

Question 48: In a DNA molecule, adenine (A) always pairs with:

a) Thymine (T)
b) Cytosine (C)
c) Guanine (G)
d) Uracil (U)

Answer: a) Thymine (T)

Question 49: What is the term for a change in the DNA sequence of a gene?

a) Mutation
b) Transcription
c) Translation
d) Replication

Answer: a) Mutation

Question 50: The process by which genetic information is copied from DNA to RNA is called:

a) Translation
b) Replication
c) Transcription
d) Transduction

Answer: c) Transcription

Question 51: Which genetic disorder is characterized by an extra copy of chromosome 21,
leading to intellectual disabilities and certain physical features?

a) Hemophilia
b) Down syndrome
c) Sickle cell anemia
d) Muscular dystrophy
Page 71 of 92
Answer: b) Down syndrome

Question 52: What is the term for the observable characteristics or traits of an organism?

a) Genotype
b) Phenotype
c) Allele
d) Mutation

Answer: b) Phenotype

Lab technician exam questions


01. Blood is which Type of Tissue?
a. Epithelial Tissue
b.Muscle tissue
c Connective Tissue✔
d.Nervous Tissue

02. Mycoplasmas are differentfrom other bacteria by?


a. Presence of chitin in cell wall
b.Presence of Techoic acid in cell wall
c.Presence of Lipoprotein in cell wall
d. Absence of cellwall itself.✔

03. The Difference of Plasma and Serum is that Plasma?


a. Dosenot contain fibrinogen.
b. has more water
c. Contain Fibrinogen.✔
d. contain Iron.

04. New Methylene Blue Reagent is used for staining of which blood cells?
a. Reticulocytes.✔
b. Platelets.

05. Gram Staining was introduce by which scientist ?


a. Alfred Gram
b. Christian Gram.✔
c. Robert Cook
d. Leuis Pasteur

Page 72 of 92
06. When WBC are counted manually by turk’s Reagent, Blood is diluted by Ratio ?
1:20
1:50
1:100
1:200

07. The end Point of Purine Metabolism is?


a. Urea
b. Creatinin
c. Uric Acid✔
d. Ammonia

08. Which is the following chemical test differentiate between Staphylococcus and Streptococcus
a. Ammoniase Test
b. Catalase Test✔
c. Urase Test
d. Oxidase Test

09. Which is the following Immunoglobulin can cross Placenta ?


A. IgA
B.IgD
c. IgG
d. IgM

10. All of the following are function of blood except one?


A. Hormone Production ✔
b. Buffer system
c. Oxygen Transport
d. Nutrient Absorption

11. Which is the following test is used to detect/determine Syphilis ?


ASOT
Ra factor
CRP
VDRL✔

12. One of the following is major cation found in extracellular fluid?


Sodium✔
Potassium

Page 73 of 92
Magnesium
Zinc

13. Immunoglobuline are which type of Protein in Nature?


Alpha Globulin
Beta Globulin
Gamma Globulin✔

14. Entrobius Vermicularis is also known as ?


Pin worm✔
Round worm
Thread Worm
Whip Worm

15. Which is the following hormone directly Regulate Blood Glucose Level?
TSH
FSH
Progesterone
Insulin✓

16. Which is the following Anticoagulant Tube need to collect sample of HbA1c ?
Sodium Flouride
EDTA✔
Heparin
Sodium Sitrate

17. Which is the following Parasite Doesn’t develop Systic stage?


Giardia Lambia
Antamoaba Histolytica
Enterobius Vermicularis
Trichomonas Vaginalis✔

18. A Blood Donor who recently diagnoses Positive HbsAg should be deffered for how long?
Permanantly✔
For 3 months
For 6 Months
For 1 Year

Page 74 of 92
19. Which is the following is called “bad” Cholestrol?
Cholestrole
Triglyceride
HDL
LDL✔

20. Which one of the following test is used to differentiate Staphylococcus aureus from other Stapl
Urase Test
Coagulase Test✔
Oxidase Test

21. Hemophilia A is caused by which factor deficiency?


Factor VIII✔
Factor IX
Factor X
Factor I

22. Salmonella and Shigella can be differentiate on the base of which test?
Gram Stain
Catalase Test
Motility✓
Indole Test

23. Ammonium Oxalate reagent is used for the counting of which test?
WBC’s
Platelets ✓
RBC’s
Reticulocytes

24. Which of the following Bio-chemistry Test Should keep away from light?
Bilirubin✔
Glucose
calcium

26. Which one of the following Immunoglobulin appeared as Pantamer?


IgA
IgE
IgG
IgM✔

Page 75 of 92
27. The solution used to fix a pap smear is:
wright stain
hematoxylin
physiological saline
cytospray✔

28. Which on of the following bacteria can cause glomerular nephritis or FR?
Staphylococcus aureus
Streptococcus Pyogenes✔
Staphylococcus Epidermidis
Streptococcus Pneumonia

29. Which one of the following hormone is produced by pituitary gland?


TRH
T4
TSH✓
Insulin

30. in DIC all of the following test will show high results except?
PT
Platelets✓
APTT
None of the above

31. Three types of ketone bodies include acetoacetic acid, acetone and __?
Oxaloacetic acid
Lactic Acid
3-hydroxy Butilic acid ✔
Acietic Acid

32. The degree that a control results deviate from known valus or mean value is known as :-
Standard Deviation✔
Coefficient Deviation
Median
None of above

33. Erythroblastosis Foetalis accurs due to:-


ABO Incompatibility
RH Incompatibility✔

Page 76 of 92
Hemophilia
Leukemia

34. One of the following enzyme also performed cardiac enzyme function?
Acid Phosphatase
Creatinine
LDH✔
ALT

35. ESR will show low value result in which one of the following condition :-
Polycythemia✔
Anemia
RA
Tuberculosis

36. Which one of the following paracite causing ameobic dysentery?


Giardia Lambia
Entamoeba Histolytica✔
Ascaris Lumbericodes
Shigella

37. Megaloblastic anaemia is caused by deficiency of folate and which vitamin?


Vitamin A
Vitamin B12✓
Vitamin D
Vitamin C

38. Which One of the Following bacteria is gram negative, Aerobic oxidase positive and motile
rod?
E-Coli
Proteus
Salmonella
Pseudomonas✔

39. Which one of the following cells play a major role in humoral immunity?
B-Cells✔
T-Cells
Monocytes
Macrophages

Page 77 of 92
40. In Iron Deficiency anemia laboratory finding which one of the following will show increased
value?
Hb
Iron
TIBC✓
RBC’s

41.Widal test is done for the diagnosis of which bacteria ?


Shigela
E-Coli
Salminella✔
Proteus

42. If Sample is taken in EDTA Tube, Which one of the following biochemistry test results will
affect?
Glucose . Urea
Cholestrol
Calcium ✔

44. Ziel Nelson stain is used for?


Streptococcus
Mycobacterium ✔
H Influenza
Bacillus sp

45. Co-efficient of variation is calculated by which formula?


Sd x 100/ Standard Error (SD-Standard Deviation)
Mean x 100/SD
SD x 100/Mean✔
Variance x 100 / Mean

46. Night Blindness is caused by deficiency of which vitamin ?


Vitamin C
Vitamin A ✔
Vitamin B12
Vitamin B6

47. What is erythropoietin ?


a. is secreted by kidney

Page 78 of 92
b. Stimulates the bone-marrow to produce RBC’s 3. Is released in responce to hypoxemia
c. All of above✔

48. Each of following characteristics is correct about Ascaris lumbricoides except one :-
a. Ascaris lumbricaid is nematode
b. Both dogs and cats are intermediate host of Ascaris lumbricoides✔
c. Ascaris lumbricoides can cause pneumonia
d. Ascaris lumbricoides is transmetted by enjections of eggs

49. Which types of cells if rapidly can cause jaundice?


WBC’s
Platelets
Plasma Cells
Red Blood Cells✔

50. Kala- Azar Disease is caused by which Parasite?


Lieshmania donovani✔
Trypanosoma Cruzi
Wucheria Bancrofti
Plasmodium

51. When myocardial infarction will happen which of the following cardiac enzyme will elevate
LDH
CK✓

52. Cason’s test is used for the diagnosis of which parasite ?


Taenasis
Hydatidasis✔
Trichurosis
OOnchuriasi

Clinical microbiology
1. Chose the gram-positive bacteria.
a. Neisseria,
b. Klebsiella,
c. Clostridium
2. Free-living, Thick wall, cause abscess of skin. Characteristics of.
a. Streptococcus

Page 79 of 92
b. Staphylococcus
c. Bacillus
3. Extracellular bacteria, thick wall, spore-forming aerobe Characteristics of.
a. Clostridium
b. Corynebacterium
c. Bacillus
4. Gram-negative cocci are.
a. Neisseria
b. Haemophilus
c. Bacillus
5. Gram-negative, rod, Facultative, straight are the Characteristics of.
a. Haemophilus
b. Neisseria
c. Yersinia
6. Respiratory gram-negative bacteria is.
a. Bordetella
b. Brucella
c. Yersinia
7. Zoonotic Organism is.
a. Pasteurella
b. boradetell
c. E.coli
8. Which bacteria cause typhoid fever.
a. Proteus
b. Salmonella
c. Shigella
9. What is the pathogenic factor of Peptic ulcer?
a. Vario
b. Campylobacter
c. Helicobactor
10. Q-fever caused by the ?
a. Chlamydia
b. Rickettsia
c. Vario
11. The causative agent of Diphtheria is.
a. listeria
b. Nocardia
c. Corynebacterium
12. Chose important normal flora of the skin.
a. Epidermidis
b. E.coli
c. Lactobacillus
13. Flexible, thin-walled cells (spirochetes), Lyme disease is the characteristics of.
a.Treponema

Page 80 of 92
b. Borrelia
c. Leptospirara
14. Normal flora is the organisms that are the.
a. Permanent resident of body
b. Came outside
c. Normal cells
15. Yeast can be normal flora.
a. True
b. false
16. Commensals are.
a. Get benefits from their host and cause damage to them
b. Get benefits from their host but did no damage to them
c. Free-living
17. Important normal flora of colon is.
A. e.coli
b. Fragilis
c. Viridin
18. An outbreak of post-surgical wound infection caused by staph aureus has
occurred in the hospital. Use your knowledge of normal flora. Which of the following
sites is the most likely location for staph aureus.
a. Throat
b. Nose
c. Mouth
19. Staph aureus is present on the skin but the main site of it is.
a. Colon
b. Nose
c. Mouth
20. Which part of the body contains the highest number of flora.
a. Mouth
b. Stomach
c. Colon

PROMETRIC mlt practice test


1. Glass material used to measure urine volumes of 24 hours:
A. Volumetric Flask
B. Beaker
C. Erlenmeyer cylinder
D. graduate cylinder
I. Safety bulb
Answer: D) Graduate cylinder

2. Glassware used for making 100 ml of 12% solution:


A. Volumetric Flask
B. Beaker

Page 81 of 92
C. Erlenmeyer cylinder
D. graduate cylinder
I. Safety bulb
Answer: a) Volumetric flash

3. A suction device for pulling fluid:


A. Volumetric Flask
B. Beaker
C. Erlenmeyer cylinder
D. graduate cylinder
I. Safety bulb
Answer: e) Safety bulb

4. The pipette is one with the middle part raised:


A. Stamp Pipette
B. Grass Land Pipette
C. Serological pipette
D. volumetric pipette
I. Micropipette
Answer: D) Volumetric pipette

5. No piece of glassware gives a significant amount of:


A. Volumetric Flask
B. Beaker
C. Erlenmeyer cylinder
D. graduate cylinder
I. Safety bulb
Answer: b) Beaker

6. Durable material used for making heat resistant glassware:


A. Polyethylene
B. Soda lime
C. Polystyrene
D. Borosilicate
I. Polyvinyl chloride
Answer: D) Borosilicate

7. Solid crystals of potassium oxalate are mixed in distilled water in a container. Potassium oxalate
is described by which term?
A. Solution
B. solvent
C. molten material
D. Regent
I. AC
Answer: c) Solvent

8. The destruction of all micro-organisms, including spores:


A. Cleanup
B. Resistance
C. Sterilization
D. disinfectant
I. deflation

Page 82 of 92
Answer: c) Sterilization

9. A unit to describe ugly:


A. Time
B. Vol
C. distance
D. Weight
I. Length
Answer: D) Weight

10. A pH of 2 is _____ compared to a pH of 5:


A. 1000 times more acidic
B. 100 times more acidic
C. 2 times less acidic
D. 20 times less acidic
I. 1000 times more acidic
Answer: e) 1000 times more acidic

11. Hypertonic solution consists of cells:


A. Swell and blast
B. Dehydration
C. Hemolaij
D. was not impressed
I. Prevent mitosis
Answer: b) Dehydration

12. The solution to cure a Pap smear is:


A. Wright’s stain
B. Hematoxylin
C. Physical Saline
D. Cytospray
I. Methylene Blue
Answer: D) Cytospray

13. Blood must be prepared for RBC calculation:


A. EDTA blood
B. seated blood
C. Heparinized blood
D. Oxalated blood
I. Blood clot
Answer: a) EDTA blood

14. What factor is not routinely used to preserve tissue in a lifestyle:


A. Formic acid
B. Zener fluid
C. 40% formaldehyde is dissolved in water
D. Beuin fluid
I. 10% formalin
Answer: a) formic acid

15. Which part of the histology equipment does not depend on temperature:
A. Wax

Page 83 of 92
Bishu processor
C
D. Embedding Center
I. Water bath
Answer: c) Microtome

16. Biopsy:
A. Expulsion of organic liquid
B. Displacement
C. Post mortem examination
D. Representative tissue pattern
I. Blood collection
Answer: D) Excision of a representative tissue sample

17. During tissue processing, what is the correct sequence of steps:


A. Clearing, dehydration, infiltration
Clearing, infiltration, dehydration
C. Dehydration, infiltration, clearing
D. Dehydration, clearing, infiltration
I. Embedding, sectioning, staining
Answer: D, dehydration, clearing, infiltration

Diagnosis is important in tissue processing because it:


A. Prevents cell morphological changes and contractions
B. Allows the tissue to be tested in a life-like state
C. Simplifies the blurring process
D. AC
I. A, B, and C.
Answer: e) a, b, c

The liquid fraction of blood remaining after coagulation is called:


A. Buffy coat
B. Serum
C. Plasma
D. lymph
I. Tissue fluid
Answer: b) Serum

20. No test can be done on the serum sample:


A. Iron
B. Vitamin B12
C. Total lipids
D. freezing factors
I. Potassium
Answer: D) freezing factor

21. The normal erythrocyte size is as follows:


A. Bi-sectional disk
B. Spirocyte
C. Palimarphonyukliyosait
D. Slim column
I. Bull’s eye

Page 84 of 92
Answer: a) Decatur disk

22. Glucose results are correct:


A. G / mmol
B. mmol / L.
C. G / L
D. G / ml
I. g / dl
Answer: b) MMOL / L.
23. If a patient refuses to have a venipuncture done you should:
A) tear up the requisition
B) collect a urine sample
C) politely ask a patient to come back next week
D) restrain the patient and proceed with the venipuncture
E) notify the patient’s physician
Answer: e) notify the patients physician
24. Which statement is incorrect when doing venipuncture:
A. The vein enters an angle of 15-20 degrees.
B. The tubes are pushed into the needle with a strong hand thumb
C. When the needle enters the bevel vein it is lifted
D. The tricyclic is removed before the needle is withdrawn
I. The patient’s arm is cleaned before the vein ruptures
Answer: e) The patient’s hand is cleaned before the vein is turned.

25. Biohazard container is used to ignore:


A. Lancets
B. needle
C. Band-Aid Rapper
D. A and B.
I. A, B and C.
Answer: D) A and B

26. Which needle gauge corresponds to the smallest needle size:


A. 18
B. 20
C. 21
D. 22
I. 23
Answer: e) 23

27. Vacuum tube used to collect and separate serum:


A. Red
B. Green
C. Lavender
D. light blue
I. SST
Answer: e) SST

28. If Lavender Top, Plane Red Top, Gray Top and Light Blue Top Tube are collected, what is the
order of the draw:
A. They can be collected in any order.
B. Plain red top, lavender, blue, gray

Page 85 of 92
C. Blue, Plain Red Top, Gray, Lavender
D. Gray, Blue, Lavender, Plain red top
I. Plain red top, light blue, lavender, gray
Answer: e) Plain red top, light blue, lavender, gray

29. The walk is flat:


A. Applied very strongly to the hand
Used to increase venous filling
C. About 6-8 above the elbow
D. tied in a knot to keep it secure
I. Needle released after withdrawal
Answer: b) Used to increase venous filling

30. If the patient falls unconscious at the time of the injury, you must:
A. Call the doctor once
B. Remove the needle and present it to the patient
C. Shout out loud
Continue this process until all blood is collected
I. Start artificial respiration immediately
Answer: b) Remove the needle and leave the patient

31. No vein / vein is used to obtain a venous blood sample:


A. The basilica vein
B. cephalic vein
C. Medial cubicle vein
D. Femoral vein
I. The veins behind the hand
Answer: D) Femoral vein

32. Blood sample collected in a heparinized tube was centrifuged. It divides:


A. Serum and clotting
B. Plasma and coagulation
C. Serum and plasma
D. Plasma, Buffy Coat, RBC
Answer: D) Plasma, Buffy Coat, RBC

33. As a result of hemolysis:


A. Using a 25-gauge needle on adults
B. The blood sample is definitely shaking
C. Cooling the void before use
D. Leave the tourniquet for 3 minutes
I. All of the above
Answer: e) All of the above

34. Examination Procedure Using Vestigrain Tube


A. Erythrocyte sedimentation rate
B. Hematocrit
C. Reticulocyte counting
D. Micromatocrit
I. Difference
Answer: a) Erythrocyte sedimentation rate

Page 86 of 92
35. Latex Gloves Protect Laboratory Worker:
A. Sudden needle puncture
B. minor injury
C. Patient Aerosol
D. Body fluid
I. All of the above
Answer: D) body fluids

36. Which statement is incorrect when setting ESR:


A. It should be read in exactly one hour
B. It should be installed near the centrifuge
C. The blood level must be zero
D. It should be done on fresh blood
I. It should be placed in a vertical position
Answer: b) It should be installed near the centrifuge

37. What is the normal temperature of a laboratory refrigerator:


A. 4C
B. 0 c
C. 3 c
d.20 c
e.37 c
Answer: c) 3 c

38. The purpose of differentiation is as follows:


A. Determine the RBC ratio in whole blood
Calculate the number of WBCs in whole blood
C. Determine the WBC ratio in whole blood
D. C and E
I. diagnose anemia
Answer: c) Determine the ratio of WBC in whole blood

39. Blend well before testing blood samples for cell count:
A. Avoid Platelets Clamp
B. Avoid small clots
C. Give oxygen to the sample
D. Also determine the distribution of all blood components
I. A mixture of anticoagulants with blood
Answer: D) Also determine the distribution of all blood components

40. Automated Hematology uses the Cell Count formula:


A. Spreading
Color absorption change
C. High Frequency
D. change in cell electric current
Answer: D

41. Maximum depth to puncture the heel on the newborn:


A. 1.5 mm.
B. 2.0 mm
C 2.4 mm

Page 87 of 92
D. 2.8 mm
I. 3.0 mm
Answer: C) 2.4 mm

42. The first drop of blood is removed after the skin is punctured:
A. Remove any existing pathogens
B. Increased blood flow to this area
C. Remove the last trace of alcohol
D. Remove any excess tissue fluid
I. C&D
Answer: e) C and D

43. Which areas are suitable for perforation of the baby:


A. Any so-called areas of the foot
B. The second or third finger on both hands
C. heel curve
D. lateral heel
Answer: D) The lateral and flat part of the heel

44. Laboratory section of antigen-antibody reaction studies what:


A. Blood
B. Gerontology
C. Immunology
D. Chemistry
I. Freezing
Answer: c) Immunology

45. Which pipe is pulled for ANA:


A. Red
B. Gray
C. Esesti
D. Green
I. Light blue
Answer: c) SST.

46. Disinfectant used on a metal surface:


A. 10% formalin
B. 2% glutaraldehyde
C. 1% hypochlorite
D. 70% isopropyl alcohol
I. 15% iodine
Answer: b) 2% glutaraldehyde

47. Which tube is collected for cross-match:


A. Lavender
B. light blue
C. Green
D. Gray
I. Plain red top
Answer: e) Plain red top

48. A special serum from dark yellow to amber is called:

Page 88 of 92
A. crenated
B. Liphemic
C. Jiggar
D. Illness
I. Hemolyzed
Answer: D) Iconic

49. Which factor disrupts the development of pathogens:


A. Appropriate nutrients
B. dark
C. humid weather
D. An acidic pH
I. 37 C. Temperature
Answer: d) An acidic pH

50. A sample:
A. Spread the material on the slide
Blood or urine volume
C. A small sample taken to represent the whole organism or system
D. a microbial colony that grows on solid medium
I. The technique used for microscopic examination of urine
Answer: c) a small sample taken to represent the whole organism or system

51. The purpose of curing bacterial scar is as follows:


A. Prevent washing of cells during stains
B. causes cells to absorb stains easily
C. Give warm temperature for bacteria to grow
D. Make the cells visible under the microscope
I. Destroy the bacterial cell wall
Answer: a) Prevents washing of cells during stains

52. Write a sample site in a swab container:


A. Warns employees about possible pathogens
B. If time permits – it is always necessary
C. Determine whether there are appropriate and weather requirements
D. determine o.h.i.p. Fees
I. A, B, C, D and E.
Answer: c) To determine the appropriate agar and weather requirements option

53. Identify the correct sequence of steps on the gram stain process:
A. Primary stain, secondary stain, mordant, decolorizing
B. mordant, primary stain, decomposition, forgery
C. Counterstain, mordant, primary stain, decolorizing
D. Primary stain, merodant, decorating, counter stain
I. None of these
Answer: d) Primary stain, mordant, decolorizing, counter stain

54. How to Store Commercially Made Culture Plates:


A. Many hours before use in the freezer
B. Reversal to prevent drip freezing in the media
C. At room temperature in the dark area of the laboratory
D. At a temperature of 2 * C – 4 * C

Page 89 of 92
I. B&D
Answer: e) b and d

55. A Gram stain acts as a valence to bind reagent stain to bacteria:


A. Lugol iodine
B. Safranin
C. Acetone-alcohol
D. gram iodine
I. Crystal Violet
Answer: D) Village iodine

56. Identify the false statement regarding blood culture collection:


A. The site is cleaned with bettin and alcohol
B. The arterial specimen is collected
C. Aerobic design is required
D. Blood culture tubes are always drawn first
I. Samples are never refrigerated
Answer: b) The arterial sample is collected

57. Sensitivity test:


A. How fast the microbiome can be destroyed
B. Identifies the type of microbe in the sample
C. determines the growth needs of organisms
D. producing a pure culture
I. Identifies the antibiotic needed to kill the microbe
Answer: e) Identifies the antibiotic needed to kill the microbe

58. Identify false statements when a sample is cultured:


A. Required equipment loop and live flame
B. The media is brought to room temperature before use
C. Depending on the type of media model selected
D. The loop is sterilized before sterilization
I. Petri lid is placed upright on the bench to prevent contamination
Answer: e) Petri lid is placed upright on the bench to prevent contamination

59. A statement is false when a gram is wet:


A. Distilled water is used for washing
B. Acetone-Alcohol reduces Gram-negative bacteria
C. Safranin turns Gram-negative bacteria into red
Gram iodine is used to bind the primary stain
I. Crystal violet violet-positive bacterial violet stain
Answer: a) Distilled water is used for washing

60. When processing the swab, body fluids or blood are most commonly used for protection:
A. Fume hood
B. Square L Laminar Flow Hood
C. Square ll Laminar Flow Hood
D. Square LL Laminar Flow Hood
I. Glove Box
Answer: C) Class L Laminar Flow Hood

Page 90 of 92
61. You are collecting blood sugar levels. The patient asks if you think he has diabetes. You’ll tell
him:
A. It is a possibility but you are not positive
B. You cannot give him any information
C. He can answer this question by discussing it with the doctor
D. You are instructed not to give any information, so you cannot discuss it
I. Read a prepared booklet and make your decision
Answer: c) Discuss this question with a therapist because he can answer this question.

62. Autoclave for small loads is set to _____:


A. 50 * 6 minutes for 121 * C.
B. 130 * C for 30 minutes.
C. 121 * C. 15 minutes 15 minutes.
75 minutes at 151.i5 D. 121 * C.
I. 154 * C 20 minutes.
Answer: C) 121 * C 15 minutes for 15 minutes.

63. Medicine is a universally accepted disinfectant for the office:


A. 2% glutaraldehyde
B. 1% hypochlorite
C. 10% formalin
D. 70% isopropyl alcohol
I. 5% iodine
Answer: b) 1% hypochlorite

64. The patient’s health card contains ___ digits:


A. 4
B. 6
C. 8
D. 10
I. 12
Answer: D) 10

65. Dilute 1/6 of serum in water. Glucose Outcome 4.0 mmol / L. What is the reported result:
A. 0.66 mmol / L.
B. 4.0 mmol / L.
C. 24.0 mmol / L.
D. 40.0 mmol / L.
I. 60.0 mmol / L.
Answer: C) 24.0 mmol / L

66. How many ml of 4% hydrochloric acid will make 100 ml of 20% hydrochloric acid:
A. 50 ml.
B. 80 ml.
C. 100 ml.
D. 500 ml.
I. 1000 ml.
Answer: D) 500 milliliters

67. How many grams of NaCl is required to make 300 ml of 2% solution:


A. 2 grams

Page 91 of 92
B. 4 grams
C. 6 g
D. 20 grams
I. Unable to identify with the information
Answer: c) 6 grams

68. How many centimeters are there in a foot:


A. 3
B. 12
C. 24
D. 30
I. 100
Answer: D) 30

69. There are two standard deviations from the mean:


A. 5% off all prices
B. 34% of all prices
C. 50% of all prices
D. 68% of all prices
I. 95% of all values
Answer: e) 95% of all values

Page 92 of 92

You might also like